You are on page 1of 49

General Nursing Board 101 Community Health Nursing Pre-test 1 http://nsgbrd101.proboards.com/index.cgi?

i?board=answers&action=display&thread=5 Community Health Nursing Pre-test 1 Post by admin on Aug 18, 2010, 12:53pm 1. This STD is characterized by greenish yellow, frothy musty odorous vaginal discharge accompanied with vaginal itchiness and painful urination? a. Trichomoniasis b. Chancroid c. Moniliasis d. Chlamydia Ans. A. Trichomoniasis 2. DOTS is a comprehensive strategy to detect and cure TB. The primary element of DOTS is a. Health workers counsel and observe their patients swallow each anti-TB medication and monitor progress until cures b. Regular drug supply c. Political will in terms of manpower and funding d. Sputum microscopy services Ans. A. The main strategy of the NTP is Directly Observed Treatment, short course (DOTS). This is a comprehensive strategy to control TB which primary health services around the world are using in the detection and cure TB 3. What is the mode of transmission of Pulmonary Tuberculosis? a. Fecal-oral c. Airborne, droplet b. Direct contact d. Blood borne Ans. C. (Martinez, D. Intensive Final Coaching Primary Health Care, Legazpi City: A1 Review Center, 06, question 36) 4. Standard examination in detecting PTb: a. X-ray c. Ultrasound b. Sputum Exam d. Tuberculin Test Ans. B. 5. In Catergory II of the TB treatment regimen all are prescribed BUT one: a. Relapse case b. New pulmonary smear (+) cases c. Failure cases d. Other (smear +) Ans. B. Category I: Prescribed for a) new pulmonary smear (+) cases, b) new seriously ill pulmonary smear(-) cases with extensive parenchymal involvement and c.) new severely ill extrapulmonary TB cases; Category II: Prescribed for a.) failure cases, b) relapse cases; c) RAD (smear +), and OTHER (smear+); Category III Prescribed for a.) new smear (-) but with minimal pulmonary TB on radiography as confirmed by medical officer and b) new extrapulmonary TB (not serious). ( 6. Malaria can be prevented by the following EXCEPT: a. Avoiding outdoor night time activities b. Use of mosquito repellants c. Planting of herbal plants which can be mosquito repellants d. Spending leisure time in the forest Ans. D. Preventive activities against malaria include all but not limited to the following: 1. Avoid going outdoor between 9pm to 3 am , the peak biting hours of female anopheles mosquitoes 2. Prophylaxis: taking chloroquine every week starting one to two weeks before traveling 3. Using mosquito net or curtain treatment by soaking the mosquito net in insecticidal solution and allow to dry before using 4. House spraying of insecticide inside the house 5. Wearing clothes that cover arms and legs at night 7. During this stage the H fever is said to cause severe abdominal pain, vomiting and frequent bleeding from

GI tract: a. Invasive Stage c. Convalescent Stage b. Toxic stage d. Febrile Stage Ans. B. Toxic Stage A. Invasive stage or febrile stage starts abruptly as high fever, abdominal pain and headache; later flushing which maybe accompanied by vomiting and epistaxis; B. Toxic or hemorrhagic stage is the time where severe abdominal pain, vomiting and frequent bleeding from the GI is observed; C. Convalescent or recovery stage most of the vital signs are stable. 8. What is the etiologic agent of Diptheria? a. Klebs-Loffler bacillus b. Bordet Gengou Bacillus c. Vibrio El Tor d. Filterable virus Ans. A. Klebs-Loffler bacillus 9. An example of this route of transmission is the dust particles with infectious agent and residue of evaporated droplets that remain suspended in the air: a. Vectorborne transmission c. Contact transmission b. Airborne transmission d. Droplet transmission Ans. Ans. B. Airborne transmission A. Vectorborne transmission examples are flies, ticks, mosquitoes and flies; B. Airborne transmission includes droplet nuclei, dust particles with infectious agent and organisms shed into the environment from the skin, hair or perineal area; C. Contact transmission includes direct contact, indirect contact and droplet contact; D. theres no such thing as droplet transmission only droplet contact. 10. This chain of infection allows the microorganisms to move from reservoir to host: a. Portal of Exit c. Portal of entry b. infectious agent d. susceptible host Ans. A. Portal of exit A. Some examples of movement from reservoir to host are excretions, secretions and skin droplets; B. The common site for the portal of entry is mucous membranes, GI tract and non-intact skin; C. Examples of infectious agents are bacteria and virus; D. susceptible host are the immunosuppressed patient, fatigued, elderly and other hospitalized patient. 11. Inflammatory processes provided by the WBC (leukocytes), whose main purpose is to limit the effect of harmful bacteria or injury by destroying or neutralizing the organism, and by limiting its spread throughout the body: a. First line of defense b. Second line of defense c. Third line of defense d. Fourth line of defense Ans. B. Second line of defense A. First line of defense include the skin and mucus membrane; B. Second line of defense involves leukocytes; C. Third line of defense include the antibodies 12. The immune response is considered to be: a. First line of defense b. Second line of defense c. Third line of defense d. Fourth line of defense Ans. C. Third line of defense A. First line of defense include the skin and mucus membrane; B. Second line of defense involves leukocytes; C. Third line of defense includes the antibodies. 13. Nurse Angelika believed that the complete destruction of all microorganisms, including the spores is:

a. Disinfection c. Cleaning b. Sterilization d. all of these Ans. B. Sterilization 14. When asked about the risk of transmitting agents through large particles (5 micrometer or more) Crizelda refer this type of precaution to her clients as: a. Airborne precaution b. Contact precaution c. Standard precaution d. Droplet precaution Ans. D. Droplet precaution 15. It is an upper respiratory condition characterized by escalating fever (peaks 3-5 days), conjunctivitis and coryza: a. Measles b. German Measles c. chicken pox d. Varicella Ans. Measles 16. A day biting female mosquito that breeds in household or standing clean water: a. Female anopheles b. Male anopheles c. Aedes aegypti d. None of these Ans. C. Aedes aegypti 17. The following are the major causes of intestinal parasitic infections in the Philippines, EXCEPT: a. Trichuris trichiura c. Hookworm b. Ascaris lumbricoides d. none of these Ans. D. All of them are the three major causes of intestinal parasitic infections 18. What is the period of communicability of the epidemic parotitis? a. Begins before the glands are swollen and presumed to last as long as localized glandular swelling remains b. Begins after the glands are swollen especially when the localized glandular swelling remains c. Begins before the glands are swollen and after the glands are swollen even when the glandular swelling is not observed. d. except c Ans. A. Begins before the glands are swollen and presumed to last as long as localized glandular swelling remains 19. Which his TRUE regarding Filariasis? a. Microfilarie rate increases with age and then levels off during the acute stage b. Men have higher micronlariae rate than women c. During the chronic stage the disease develops 2-5 years from the onset of attack d. Hydrocoele is evident in men during the acute stage A. It is not during the acute stage but during the asymptomatic stage: B True that men have higher micronlariae rate than women; C. The disease develops from 10-15 years from the onset of attack; D. It is during the chronic stage when hydrocoele will be noted. 20. Which is FALSE about Tuberculosis? a. The most hazardous period for development of clinical disease is the first 6-12 months after infection b. The risk of developing the disease is highest in children under 3 years old. c. The degree of communicability depends on the virulence of the bacilli, adequacy of ventilation with no direct exposure of bacilli to the sun or UV light d. Susceptibility of the disease is markedly increased in those with HIV infection and other forms of immunosuppression Ans. C. All are true except C because the degree of communicability always involves exposure of the bacilli to the sun or UV light. 21. Which of the following tetanus immunization is given to a mother to prevent from a neonatal tetanus and provide 10 years protection for the mother? a. TT1 b.TT2

c. TT3 d. TT4 Ans. C. Schedule of TT immunization: VACCINATION SCHEDULE PROTECTION DURATION OF PROTECTION TT1 As early as possible during pregnancy 80% TT2 4 weeks after TT1 80% Infants protected from neonatal tetanus. Provides 3 year protection to the mother TT3 6 months after TT2 90% Infants protected from neonatal tetanus. Provides 5 year protection to the mother TT4 1 year after TT3 99% Infants protected from neonatal tetanus. Provides 10 year protection to the mother TT5 1 year after TT4 99% Infants protected from neonatal tetanus. Provides lifetime protection to the mother.

22. Which of the following vaccines is not recommended by IM injection? a. Measles vaccine c. Hepa B vaccine b. Tetanus toxoid d. DPT Ans. A. Measles vaccine 23. The following are vaccines which are less sensitive to heat: a. oral polio, DPT b. Measles, Hep B c. BCG, TT d. BCG, oral polio Ans. C. BCG and TT Most sensitive to heat: oral polio and measles; least sensitive to heat: DPT, Hep B, BCG, TT

24. OPV should be stored in a freezer with a temperature of: a. 2-8 degrees C b. -15 to -25 degrees C c. -2 to -8 degrees C d. none of these Ans. B. -15 to -25 degrees C 25. Hepa B is given at birth with 6 weeks interval from the 1st dose to 2nd dose, 8 weeks interval from the 2nd to 3rd dose. It is given intramuscularly in the: a. outer part of upper arm b. gluteus maximus c. vastus lateralis d. deltoid region Ans. C. vastus lateralis 26. It is the principle practiced in order to assure that all vaccines are utilized before its due date: a. First expiry and first out b. First used and first contained c. First expiry and last out d. First used and last contained Ans. A. First expiry and First out 27. When reconstituting the freeze dried BCG vaccine the nurse must always keep the diluents in the room temperature by sustaining with the BCG vaccine ampules in the clinic table: a. TRUE b. FALSE c. TRUE in Hep B and DPT d. except A Ans. A. FALSE The BCG vaccine must always be kept in cold temperature and sustaining it in the refrigerator or vaccine carrier. 28. What is the classification of disease when the level of management is urgent referral in hospital?

a. mild b. moderate c. severe d. most severe Ans. C. Severe. 29. What is the color presentation to children whose level of management is home care. a. yellow b. red c. green d. pink Ans. C. Green. ( 30. The following BUT one is part of the danger signs that needs to be checked in assessing the child using the integrated case management process? a. vomits everything b. convulsions c. unable to drink/breastfeed d. difficulty breathing Ans. D. Difficulty of breathing is part of the main symptoms that needs to be assessed not the danger signs. The missing danger sign is abnormality sleepy or difficult to awaken.

General Nursing Board 101 Community Health Nursing Comprehesive http://nsgbrd101.proboards.com/index.cgi?board=answers&action=display&thread=7 Community Health Nursing Comprehesive Post by admin on Aug 18, 2010, 1:01pm

1. The nurse learned that the health of individuals and communities are, to a large extent, affected by a combination of many factors. She understands that these are the determinants of health listed by the World Health Organization, EXCEPT: a. Greater support from families, friends and communities are linked to better health. b. Access and use of services that prevent and treat disease influence health. c. Research, development and implementation of innovative public health solutions is also part d. Customs and tradition, and the beliefs of the family and community is also part Ans. C. Is not part of the determinants of health rather it is one of the functions of public health A. Social support system is part of the determinants of health; B. Health services is also part of the determinants of health; C. Is not part of the determinants of health rather it is one of the functions of public health; D. Culture is part of the determinants of health. ( 2. The classic definition of public health is the science and the art of preventing disease, prolonging life, promoting health and efficiency through organizing community effort for the sanitation of the environment, control of CD, education of personal hygiene, organization of medical and nursing services for the early diagnostic and treatment of disease and the development of the social machinery to ensure everyone a standard of living adequate for the maintenance of health, so organize to enable every citizen to realize his birthright of health and longevity. This definition was coined form: a. WHO Expert Committee on Nursing c. Ruth Freeman b. Dr. C.E. Winslow d. Public Health Nurse Lilian Wald Ans. B. Dr. C.E. Winslow A. WHO defined public health as the art of applying science in the context of politics so as to reduce inequities in health while ensuring the best health for the greatest number; B. Public health defined by Dr. Winslow exactly as mentioned above; C. Freeman defined community health nursing not public health as a service rendered by a professional nurse with communities, groups, familiesfor the promotion of health, prevention of illness, care of the sick at home and rehabilitation; D. Lilian Wald defined public health nursing

not public health as service to all the people. (Cuevas, Frances Prescilla. Public Health Nursing in the Philippines. 10th ed. Philippines: Publication Committee of NLPGN,Inc., 2007 p. 4-7) 3. The Health Care Delivery System involves two major players; the first is financed through a tax-based budgeting system and the other is largely market-oriented. They are known to be as? a. National and Local Government Agencies b. For-profit and Non-profit health providers c. Public sector and LGU d. Department of Health and Non-profit health providers Ans. D. Department of Health and Non-profit health providers (Public and Private Sector) A. The National and Local Government Agencies are part of the public sector; B. For-profit and Non-profit health providers are both part of the private sector; C. The public sector and LGU are the same that took part in the public sector. D. The department of health is a public sector while the Non-profit health provider is part of the private sector. The 2 major players in the Health Care Delivery System are the Public and Private sectors. (Cuevas, Frances Prescilla. Public Health Nursing in the Philippines. 10th ed. Philippines: Publication Committee of NLPGN,Inc., 2007 p. 19-20) 4. What is the vision of the Department of Health? a. FOURmula ONE for health b. Health Sector Reform Agenda c. Health for all in the Philippines d. Guarantee equitable, sustainable and quality health for all Filipinos, especially the poor and shall lead the quest for excellence in health Ans. C. Health for all in the Philippines A. FOURmula One for health is the framework for implementation of Health Sector Reform Agenda (HSRA); B. Health Sector Reform Agenda is the goal of the DOH; C. Health for all in the Philippines is the vision of the Department of Health; D. This is the Mission of the DOH. (Cuevas, Frances Prescilla. Public Health Nursing in the Philippines. 10th ed. Philippines: Publication Committee of NLPGN,Inc., 2007 p. 25-26) 5. The framework for the implementation of health sector reform agenda of the department of health is: a. FOURmula ONE for health b. Health Sector Reform Agenda c. Health for all in the Philippines d. Guarantee equitable, sustainable and quality health for all Filipinos, especially the poor and shall lead the quest for excellence in health Ans. A. FOURmula ONE for health A. FOURmula One for health is the framework for implementation of Health Sector Reform Agenda (HSRA); B. Health Sector Reform Agenda is the goal of the DOH; C. Health for all in the Philippines is the vision of the Department of Health; D. Guarantee . is the Mission of the DOH. (Cuevas, Frances Prescilla. Public Health Nursing in the Philippines. 10th ed. Philippines: Publication Committee of NLPGN,Inc., 2007 p. 25-26) 6. The overriding goals of the Department of Health: a. FOURmula ONE for health b. Health Sector Reform Agenda c. Health for all in the Philippines d. Guarantee equitable, sustainable and quality health for all Filipinos, especially the poor and shall lead the quest for excellence in health Ans. B. Health Sector Reform Agenda A. FOURmula One for health is the framework for implementation of Health Sector Reform Agenda (HSRA); B. Health Sector Reform Agenda is the goal of the DOH; C. Health for all in the Philippines is the vision of the Department of Health; D. Guarantee . is the Mission of the DOH. (Cuevas, Frances Prescilla. Public Health Nursing in the Philippines. 10th ed. Philippines: Publication Committee of NLPGN,Inc., 2007 p. 25-26) 7. The goal of this element in FOURmula ONE is to ensure the quality and affordability of health goods and services. a. Health financing c. Health regulation b. Health service delivery d. Good governance Ans. C. Health regulation A. Health financings goal of this health reform area is to foster greater, better and sustained investments in health; B. Health service deliverys goal is to improve and ensure the accessibility and availability of the

basic and essential health care in both public and private facilities and services; C. Health regulation ensure the quality and affordability of health goods and services; D. Good governance help enhance the health system performance at the national and local levels. Cuevas, Frances Prescilla. Public Health Nursing in the Philippines. 10th ed. Philippines: Publication Committee of NLPGN,Inc., 2007 p. 26) 8. The first international conference of Primary Health Care was held in Alma Ata, USSR on September 6-12, 1978 by WHO. The goal was adopted in the Philipppines on October 19, 1979. What is its underlying theme? a. Health in the Hands of the People by 2020 b. Health for All by the year 2000 c. Health for all in the Philippines d. Health Sector Reform Agenda Ans. A. Health in the Hands of the People by 2020 A. Health in the Hands of the People by 2020 is the underlying theme in the Philippines; B. Health for All by the year 2000 is the goal during the conference; C. Health for all in the Philippines is the vision of the Department of Health; D. Health Sector Reform Agenda is the goal of the DOH 9. The primary components of the PHC include: a. Immunization and Control Communicable Diseases b. Health Education c. Environmental Sanitation d. All of the above Ans. D. All of the above The elements or components of PHC include: Environmental Sanitation; Control of Communicable diseases; Immunizations; Health Education; Maternal and Child Health and Family Planning; Adequate Food and Proper Nutrition; Provision of Medical Care and Emergency Treatment; Treatment of Locally Endemic Diseases and Provision of Essential Drugs. 10. Mark is a trained community worker in the community. He is identified at what level of primary health care worker? a. BHWs b. Volunteer c. Intermediate level health care d. Trained Hilot Ans. A. There are two levels of primary health care workers in the community. They are: 1. Village or Barangay Health Workers who are the trained community health workers, health auxiliary volunteers, TBSs and trained healers and 2. Intermediate Level Health Workers such as the general medical practitioner, PHN, midwife, rural sanitary inspector and midwife. 11. 10. There are 4 cornerstones/pillars in PHC. Of the following which is NOT included? a. Government Funds b. Intra and intersectoral linkages c. Use of appropriate technology d. Active community participation Ans. A. Government Funds A. Government Funds is not part of the pillars of PHC; B. Intra and Intersecoral linkages is part of the 4 pillars; C. The use of appropriate technology is also part; D. Active community participation is also part of the PHC cornerstone. The missing pillar is Support Mechanism. 12. The intention of this level of prevention is to halt the disease or injury process and assist the person in obtaining an optimal status according to his present capabilities: a. Secondary Level b. Tertiary Level c. Primary Level d. Quaternary Level Ans. B. Tertiary Level A. Secondary level: Early diagnosis, detection, screening, case finding, and prompt treatment. Examples are Sputum exam, cataract screening, ORESOL , etc.; B. Tertiary level: Rehabilitation. Services provided after a disease or disability has occurred and the recovery process has begun. Ex. Community based rehabilitation program; C. Primary level: Health promotion and specific protection, seeks to prevent a disease or condition before the person gets sick. To encourage optimal health and to increase the persons resistance to illness examples are prenatal care; D. There is no such thing as quaternary level.

13. The PHN understands that this function is the most inherent function. Her practice as a nurse is based on the science and art of caring, in whatever setting she maybe or role she may presume. a. Collaborating and coordinating function b. Supervisory function c. Health promotion and Education function d. Nursing care function Ans. D. Nursing Care function A. Collaborating and coordinating function bring the PHN its activities systematically into proper relation or harmony with each other; B. Supervisory function supervises the midwives and other auxiliary health workers in the catchment area. C. Health Promotion and Education function tells that her activities go beyond health teachings and health information campaign; D. Nursing care function is the inherent function, as such PHNs are expected to provide nursing care. 14. The nursing process is systematic, scientific, and dynamic, on going interpersonal process in which the nurse and the clients are viewed as system with each affecting the other and both being affected by the factors within the behavior. The following are part of the Planning Nursing Action, EXCEPT: a. Develop evaluation parameters b. Identify needed alterations c. Prioritize needs d. Construct action and Operational plan Ans. B. Identify needed alterations A. Developing evaluation parameter is part of planning nursing action; B. Identifying needed alterations is part of Evaluation of care and services rendered; C. Prioritize needs is in planning nursing action; D. Constructing action and Operational plan is also part of the planning nursing action phase. 15. Maita a student nurse when asked by her clinical instructor the basic principles of health education responded INCORRECTLY when she mentioned that health education is: a. a creative process b. achieved by doing c. recognized as a basic function of the people d. helping people attain their health through their efforts Ans. C. Recognize as a basic function of the people A. a creative process is part of the basic principles of health education as well as B. achieved by doing; C. is wrong because it should be the primary function of the public health workers not the people; D. is correct also because it emphasizes the own effort of the people. 16. The primary focus of community health nursing is health promotion. What is its ultimate goal? a. worth and dignity of man b. making the community health nurses as generalist c. raise the level of health of the citizenry d. provide quality nursing services to individuals, families and communities utilizing as basis the standards set community health nursing practice. Ans. C. raise the level of health of the citizenry A. worth and dignity of man is the philosophy of CHN; B. making the community health nurses as generalist in the nursing profession; C. raising the level of health of citizenry is the ultimate goal; D. Provide quality nursing. Is one of the objectives. 17. These are the following principles of Community health nursing, EXCEPT: a. In CHN, the family is the unit service b. The CHNurse is responsible for her own professional growth. c. The CHNurse utilizes the already-existing active organized groups in the community. d. Opportunities for continuing staff education programs for nurses are the CHNurses responsibility. Ans. D. Opportunities for continuing staff education programs for nurses are the CHNurses responsibility A,B and C are part of the principles of Community Health Nursing. 18. The primary group focus of community health nursing practice especially in terms of maintaining the peoples optimum level of functioning is: a. Health Promotion b. Rehabilitation c. Reduction of lifestyle diseases d. Prevention of communicable disease

Ans. A. Health Promotion Although letters b, c and d are also functions of community health nursing, health promotion is the primary group focus of community health nursing practice especially in terms of maintaining the peoples OLOF. 19. These are the steps in conducting home visit. Arrange in order of priority: I. Put the bag in a convenient place then proceed to perform the bag technique II. Record all important data, observation and care rendered III. State the purpose of the visit IV. Perform the nursing care needed and give health teachings V. Greet the patient and introduce yourself a. III,V,I,II, and V b. V,III,I,IV and II c. III,V,IV,I and II d. V,I,III,IV and II Ans. B. V,III,I,IV and II. Steps in conducting home visits: 1. Greet the patient and introduce self; 2. State the purpose of the visit; 3. Observe the patient and determine the health needs; 4. Put the bag in a convenient place then proceed to perform the bag technique; 5 Perform the nursing care needed and give health teachings; 6. Record all important data, observation and care rendered and 7. Make appointment for a return visit. 20. In the Philippine Health Care Delivery System, the secondary level of health service is provided by the: a. Regional Medical Center c. Provincial hospitals b. Puericulture Centers d. Rural Health Units Ans. C. Provincial hospitals The RHCDS provides three levels of health services: a. Primary Level: BHS, RHU, Community Hospitals, Lying in centers, puericulture centers, clinics of private practitioner b. Secondary Level: Emergency hospitals, District Hospitals, Provincials and City Hospitals c. Tertiary Level: Regional Hospitals, National Medical Centers and Training Hospitals 21. The devolution of health services to the local government unit was mandated by: a. R.A. 7160 b. E.O. 851 c. R.A. 6713 d. R.A. 8749 Ans. A. RA 7160 is the local autonomy code, which transferred local health services under the authority of the local governments. EO 851 is the law of the Reorganization of DOH. RA 6713 is the code of conduct and ethical standards of public officials and employees. AR 8749 is the Clean Air Act, Approved in 2000 and took effect in 2001. 22. Nurse Cara learned that bag technique is a tool by which the nurse, during her visit enabled her to perform a nursing procedure with ease and deftness, to save time and effort in rendering nursing care to clients. Which of the following equipments are carried separately? a. Spirit of ammonia, Zephiran solution and 70% alcoohol b. Sphygmomanometer, Thermometer and Stethoscope c. Sphygmomanometer only d. Stethoscope and sphygmomanometer Ans. D. Stethoscope and sphygmomanometer A. Spirit of ammonia et als. Are carried in the bag; B. Thermometer is carried in the bag; C. Lacking stethoscope; D. Both are carried separately 23. The basic qualification of a CH Nurse is: a. Masters of Public Health b. Bachelor of Science in Nursing b. Licensed Practical Nurse d. b only Ans. D. b only 24. A process of analyzing and determining the community health status is community diagnosis. These are the following demographic variables needed in the diagnosis, EXCEPT: a. age and sex composition c. pattern of migration b. educational level d. total population Ans. B. Educational level A, C and D are all demographic variables; B. educational level is part of socio-economic and cultural variable.

25. The nurse gathered data that will be used in community diagnosis. She assessed the physical/geographical characteristics. In what factor does this belong? a. Socioeconomic and cultural variable b. Environmental Indicator c. Demographic data d. both B and D Ans. B. Environmental Indicator 26. What level of water source where a system with a source is observed, there is reservoir and piped distribution network up to the household. a. Level I b. Level II c. Level III d. Level IV Ans. C. Level III A. Level I- a protected well or developed spring with outlet but without distribution; B. Level II a system composed of source, a reservoir and piped distribution network and communal faucet located not more than 2 meter from the nearest household; C. there is piped distribution network up to the household; D. no Level IV 27. Water system and deep well construction require the approval of: a. Mayors permit b. Secretary of health c. Rural sanitary inspector d. City health engineer Ans. B. The certification of the potability of an existing water source is also issued by the secretary of the DOH. 28. Which of the following is level 1 approved type of toilet facility? a. Water sealed b. Flushed type c. Pit latrines d. Water carriage type Ans. C. Level 1: non water carriage such as pit latrines, reed odorless earth closet; Level II water carriage type such as water sealed and flushed type septic tank; Level III water carriage type connected to septic tanks and sewerage system. 29. Vital Statistics refers to the systematic study of vital events such as births, illnesses, marriages, divorce, separation and deaths. It is the measure of the risk of dying from the cause related pregnancy, child birth and puerperium. a. Infant Mortality Rate b. Maternal Mortality Rate c. Maternal Morbidity rate d. Specific Death Rate Ans. B. Maternal Mortality Rate 30. In vital statistics, this measures the frequency of occurrence of the phenomenon during a given period of time. a. Prevalence Rate b. Incidence Rate c. Specific Rate d. except c Ans. B. Incidence Rate A. Prevalence Rate measures the proportion of the population which exhibits particular disease at a particular time; B. is the correct answer; C. Specific Rate the relationship is for a specific population class or group. It limits the occurrence of the event to the portion of the population definitely exposed to it; D. only Incidence Rate or letter B 31. This is the phase of COPAR where nurse do an ocular survey of short listed community. a. Pre-entry c. Organization Building b. Entry d. Sustenance and Strengthening Ans. A. Pre-entry

32. What part of the pre-entry phase where the nurse pay courtesy call to the community leaders, sensitize community leaders and conduct baseline survey? a. Community preparation selection c. Coordination b. Final preparation d. Community preparation survey Ans. B. Final Preparation 33. When the Core group is selected the phase of COPAR is said to be: a. Phase I c. Phase II b. Phase III d. Phase IV Ans. C. Phase II 34. These are ways of partnership in COPAR, EXCEPT: a. Networking c. Collaboration b. Competition d. Cooperation Ans. B. Competition 35. The fundamental unit of any society, composed of father, mother and children related by blood or marriage is the family. When the authority inside the family is 50-50 basis or there is sharing of decision. This is said to be: a. Matriarchal c. Egalitarian b. Patriarchal d. Equalitarian Ans. C. Egalitarian A. Matriarchal mother is the dominant figure of authority; B. Patriarchal father is the dominant figure of authority; C. Egalitarian 50-50 basis, sharing of decision making; D. There is no such thing as Equalitarian 36. When the health status of the family is measured by nutritional status weight, height, mid-upper arm circumference. It is called: a. Nutritional assessment c. Dietary History b. Arthropometric assessment d. Eating/feeding habits, practices Ans. B. Arthropometric assessment A. Nutritional assessment especially for vulnerable or at risk members; B. Arthropometric assessment is the correct answer; C. Dietary history specifying the quality and quantity of food/nutrient intake per day; 37. It is the term used for intermittent occurrence of disease in a few and unrelated cases within a given locality. a. Endemic occurrence c. Epidemic occurrence b. Sporadic occurrence d. Pandemic occurrence Ans. B. Sporadic occurrence 38. Mortality and morbidity are categorized as community health nursing problems on: a. health resources c. health-related b. health status d. health indicators Ans. B. Vital Statistics is the collection of data of significant events that occur over a period of time within a population. Morbidity and mortality are among of the significant data needed to determine the health status of the community. 39. What is the leading cause of mortality in the Philippines as of 2003 statistics? a. Malignant Neoplasm c. Heart Diseases b. Vascular System Diseases d. Tuberculosis, all forms Ans. C. Heart Disease Mortality Statistics (2003) 1. Heart Diseases 2. Vascular System Diseases 3. Malignant Neoplasm 4. Accidents 5. Pneumonia 6. Tuberculosis, all forms 40. The nurse is well versed in environmental health and sanitation. She understands that the components and factors in the prevention of illness lie in the following, EXCEPT:

a. environment b. disease agent c. food safety practices d. man Ans. C. Food safety practices is not part 41. What would you include in your health teaching in iodine deficiency? a. Eat sea foods and vegetables b. Eat beans c. Eat meats and vegetables d. Eat sweet potatoes Ans. A. Good source of iodine include sea foods and iodized salt( 42. A traditional plant used to lower the uric acid. a. Pansit pansitan b. Lagundi c. Bayabas d. Sambong Ans. A Pansit pansitan , also known as ulasimang bato (Peperonia pellucida) is a herbal medicine which is known to lower uric acid and beneficial in persons with rheumatism and gout. Its leaves 1 cups leaves to two glassful of water are boiled and the decoction is taken three times a day. Fresh leaves can also be eaten as a salad three times a day. 43. A traditional plant use to treat diarrhea: a. Lagundi b. Bayabas c. Pansit pansitan d. Sambong Ans. B. Bayabas or guava (Psidium guajava) is a fruit bearing tree. Its leaves are boiled for 15 minutes at low fire and decoction. It can be used for the following medicinal purposes: washing, diarrhea, mouth gargle and relief of tooth ache. 44. An alternative for mefenamic acid for toothache is: a. Tsaang gubat b. Lagundi c. Sambong d. Bawang Ans. D. Bawang or ajos is pounded and applied to the aching tooth. To lower cholesterol and for hypertension, 2 pcs. Bawang may be fried, roasted, soaked in vinegar or boiled and taken three times a day. 45. Used to relieve muscle and joint pain: a. Lagundi b. Akapulko c. Yerba Buena d. Sambong Ans. C Yerba Buena is indicative for the following ailments: Headache, stomachache, cough and colds, arthritis causing joint pains, swollen gums, toothache, etc. 46. In the principle of from cleanest to the dirtiest care. Which of the following is arranged correctly? a. measles case, pregnant woman, newborn b. post-partum, newborn, measles case c. tuberculosis, hypertension, measles d. post-partum, measles case, newborn Ans. B. Post-partum, newborn, measles care 47. To disseminate information, the BEST type of nurse-family contact is: a. home visit b. clinic visit c. community assembly d. except b&c Ans. C. community assembly 48. The type of public health nursing that focuses on the promotion of health and wellness of pupils, teaching and non teaching personnel of the school. a. Occupational Health Nursing b. School Nursing c. Mental Health Nursing

d. All of the above Ans. B. School Nursing 49. This type of vaccines are the most sensitive to heat. They are in the form of live attenuated and freeze dried: a. DPT, Hep B b. BCG, Tetanus Toxoid c. Measles, Oral Polio d. BCG, Measles Ans. C. Oral Polio Vaccine and Measles were the most sensitive to heat with storage temperature of -15 to 25 degree C at the freezer. 50. In DPT the Diptheria and Tetanus are in the form of weakened toxin, what is the form of Pertussis? a. Freeze dried bacteria b. Live attenuated c. Weakened toxoid d. Killed bacteria Ans. D. Killed bacteria. Oral Polio is live attenuated. Measles and BCG are freeze dried bacteria. Diptheria and Tetanus are weakened toxin. 51. It helps the nurse to explain the probable cause of health conditions that occur in the community: a. Vital Statistics b. Demography c. Epidemiology d. Multiple causation theory Ans. C. Epidemiology. Vital Statistics are indices of health and illness status of the community; Demography describes the characteristics of the population in terms of size, composition and distribution in space. Multiple causation theory explains disease as caused not by single condition but by several conditions. 52. It is the intrinsic property of microorganism to survive and multiply in the environment to produce disease. a. Agent c. Causative Agent b. Host d. Environment Ans. A. Agent A. is the correct answer; B. Host influences exposure, susceptibility or response to agents; C. Causative agent is the infectious agent or its toxic component that is transmitted from the source of infection; D. Environment is the sum total of all external condition 53. The following are the components of the environment, EXCEPT: a. Physical c. Spiritual b. Biological d. Socio-economic Ans. C. Spiritual A. Physical environment is composed of the inanimate surroundings such as the geophysical conditions of the climate; B. Biological environment makes up the living things around us such as plants and animal life; C. There is no such thing as spiritual environment; D. Socio-economic environment which may be in the form of level of economic development of the community, presence of social disruptions and the like. 54. Marco was bitten by a dog. You interviewed Marco and his father to take the history before seeing a doctor. You told Marcos father not to kill the dog because the dog will be: a. Given a vaccine b. Confined c. Observed for 10 days d. Be examined Ans. C. When a person is bitten by a dog, the animal should be observed for 10 to 14 days. 1. The person bitten is immunized with rabies immunoglobulin and tetanus toxoid right after the incident to provide immediate protection; 2. If the dog dies before the the observation period is completed, the bitten person should be immunized with rabies vaccine; and 3. If the dog shows signs of rabies, the person should be immunized with rabies vaccineand the dog should be killed immediately and its brain be examined for the presence of Negri bodies, the diagnostic sign of the disease. 55. In patient with dengue fever, which of the following will you give to the patient as part of your intervention?

a. Water and salt solution b. Oresol c. Saline solution d. IV fluids Ans. B. H-Fever is characterized by internal bleeding that could result in hypovolemia so rapid fluid replacement is the most important part of the treatment. ORESOL at 75ml/kg in 4-6 hours. Up to 2-3 liters can be given in adult clients. 56. What is the test that confirms the diagnosis of Dengue Fever? a. Torniquet Test b. Capillary Fragility Test c. Platelet Count d. Rumpel Lead Test Ans. C. 57. In doing a tourniquet test, how would petechiae be considered positive? a. 10 b, 20 c. 30 d. 5 Ans. B. The tourniquet test for H-Fever is also known as Rumpel Leads Test. The procedure in taking the test is: 1. Take BP 2. Inflate the BP cuff midway between the systolic and diastolic pressure and leave for 5 minutes 3. Release cuff 4. Count then umber of petechiae below the cuff at the atecubital fossa that would fit inside a 2.5cm square of 1 inch square. The test is positive if thepatient develops 20 or more petechiae with in the area 58. The communicable disease that causes infertility in both male and female? a. Gonorrhea c. Herpes b. Syphilis d. Chlamydia Ans. A 59. Which of the following is the most common and highly contagious STD? a. Gonorrhea c. Herpes b. Syphilis d. Chlamydia Ans. D 60. Which STD causes oral thrush? a. Trichomoniasis c. Chlamydia b. Candidiasis d. Gonorrhea Ans. B

General Nursing Board 101 Maternal and Child Nursing Pre-Test 1 http://nsgbrd101.proboards.com/index.cgi?board=answers&action=display&thread=8 Maternal and Child Nursing Pre-Test 1 Post by admin on Aug 18, 2010, 1:02pm 1. A 22-year-old multigravida at 22 weeks gestation complains during routine prenatal visit of a 5-lb weight gain and vaginal discharge. Her assessment included a normal speculum examination. The nurse examines the patients abdomen and finds that the tip of the uterus is 20 cm above the symphysis pubis. This upper rounded portion of the uterus is the: a. Corpus b. Isthmus c. Fundus d. Bladder Ans: C -a nurse examines a patient at 22 weeks gestation and finds that the tip of the uterus is 20 cm

above the symphysis pubis. The upper portion of the uterus is the fundus. These options are incorrect as seen in rationale (C). 2. A patient is pregnant for the 3rd time. She has a 2-year-old son and had spontaneous abortion at 10 weeks gestation. The correct gravida and para for this patient is: a. Gravida II, para I b. Gravida II, para II c. Gravida III, para I d. Gravida III, para II Ans: C - a patient is pregnant for the third time; therefore, she is gravida III. Having a 2-year-old son counts as a para. Having had a spontaneous or elective abortion (less than 24 weeks) does not cunt as a para; therefore, she is para I. These options are incorrect as seen in rationale (C). 3. Upon pelvic examination, it was noted that a patients cervix is a bluish-purple color. This is known as: a. Goodells sign b. Braxton Hicks contraction c. Chadwicks sign d. Homans sign Ans: C -when the nurse performed a pelvic examination on the patient, the nurse noted that the cervix was bluish-purple color, which is known as Chadwicks sign. This occurs from about from the fourth week of pregnancy and is caused by increased vascularity of the vagina. Goodells sign (softening of the cervix), a probable sign of pregnancy, occur during the second month. Braxtons Hicks contractions (sign) are mild, irregular, painless uterine contractions that occur more frequently in late pregnancy. These contractions do not represent true labor. Homans sign is an early of phenothrombosis of the deep veins of the calf in which the patient complains of pain when the leg extended and the foot is dorsiflexed. 4. The active and latent phase of labor is stage: a. One b. Two c. Three d. Four Ans: A -the latent and active phase of labor, as well as the transitional phase, occur in stage one. During latent phase, the cervix dilates to 3 cm and effaces to 40 percent. During the active phase, the cervix dilates to 10 cm (complete) with 100 percent effacement, and the station increases to approximately 1 to + 1. Note: stage one lasts from the onset of regular uterine contractions to the full dilation of the cervix. These options are incorrect as seen in rationale (A), 5. The stage that begins after the delivery of the newborn is stage: a. One b. Two c. Three d. Four Ans: C - the stage that begins after the delivery of the newborn is stage three of labor. The third stage of labor. The third stage of labor lasts from delivery of the fetus to delivery of the placenta. Stage one is explained in answer 6 rationale (A). the second stage of labor lasts from full dilation of the cervix to delivery of the fetus and is generally shorter in multimanic status is observed closely. 6. Which one of the following hormones most likely inhibits uterine contractions throughout pregnancy? a. Progesterone b. Prostaglandin c. Estrogen d. Oxytocin Ans: A - progesterone inhibits uterine contractions throughout pregnancy. Prograstaglandin plays a role in ovulation, formation of corpus luteum, uterine contractility, and milk ejection prostaglandins are used to induce labor in second-trimester abortions and may be used to ripen (or prepare) a cervix for Pitocin (oxytocin) induction of labor. Estrogen is produced by the ovary and adrenal cortex in a prepregnant state; however, the principal source during pregnancy is the placenta. Estrogen increases from the onset of pregnancyis the placenta. Estrogen increases from the onset of pregnancy to term. Oxytocin is produced by the posterior lobe of the pituitary gland and causes uterine contractions. 7. A condition that predisposes a patient to postpartum hemorrhage is: a. Twin pregnancy

b. Breech presentation c. Premature rupture of membranes d. Cesarean birth Ans: A - twin pregnancy or any condition that overdistends the uterus will predipose a patient to postpartum hemorrhage.overdistention of the uterus causes poor uterine muscle tone, which inturn causes poor post partum uterine contractions, leading to an increased risk of postpartum hemorrhage. Breech presentation, in which the buttocks, feet, or both are nearest the cervical opening and are delivered first, is the most common exmale of malpresentation. The major concern in breech presentation is cord prolapse andhead entrapment. Premature or preterm rupture of the membranes will not cause postpartum hemorrhage. With preterm or prolonged rupture of membranes, there may be a danger of infection. Cesarean birth does not increase the incidence of postpartum hemorrhage. 8. During the fourth stage of labor, the mother asks the nurse, Why are you pressing on my uterus? It hurts. The nurse best response is: a. We massage every mothers uterus after she delivers. b. I know it hurts, but it has to be done. c. If I dont massage your uterus, you will not pass the placenta, and then you may hemorrhage. d. I am checking your uterus frequently to make sure it stays firm, so you wont bleed excessively. Ans: D - after the baby is born, the nurse needs to massage the patient uterus gently to encourage it to contract, and check it frequently to be sure the uterus stays firm to decrease blood loss. Telling a patient, We massage the uterus of every patent after they deliver, does not indicative to the patient the rationale for why the message must be done. Telling the patient, I know it hurts, but it has to be done, is not the reason it is done. The patient will more readily accept the uncomfortable feeling of having her uterus massaged if she knows it may prevent her from hemorrhaging. The physician makes sure the placenta is passed or delivered. 9. Shortly after delivery, the nurse assesses the new mothers uterus. It is firm and one fingerbreadth above the umbilicus and is displaced to the left of the abdomen. The nurses first priority is: a. Encourage mother to void b. Administer an oxytoxic drug c. Administer a tocolytic drug d. Vigorously massage the mothers uterus Ans: A - if, shortly after delivery, the patients uterus is firm and one finger above the umbilicus and is displaced slightly to the left of the abdomen, the nurse needs to encourage the patient to void. A full bladder is most likely causing displacement of the uterus. These options are incorrect as seen in rationale (A). 10. A patient is admitted to the obstetric floor with a diagnosis of abruptio placenta. With this complication, blood loss usually is: a. Seldom present b. Minimal c. Less than observed d. Greater the observed Ans: D -blood loss with abruption placenta is greater than observed because some blood remains in the uterus. These options are incorrect as seen in rationale (D). 11. A patient is given Methergine (methyllergonovine) 2 mg IM, 4 hours post delivery because she is experiencing postpartum bleeding. The nurse should assess for: a. blood pressure variations b. severe fluid retention c. severe hyperglycemia d. uterine rupture Ans: A - a patient receiving Methergene for postpartum bleeding should be assessed for hypertension. Methergene causes vasoconstriction and stimulates uterine contractions, which decrease bleeding. Side effects include headache, dizziness, chest pain, and hypertension. A drop in blood pressure may indicative hypovolemia. These are not side effects of Methergene. 12. During a pelvic examination. The physician notes a definite softening of the lower uterine segment, discoloration of the mucous membranes of the vagina, and softening of the cervix. These signs are referred to respectively as: a. hegars; chadwicks goodells b. goodells; chadwicks; hegars c. chadwicks; goodells; hegars

d. none of the above Ans: A - a definite softening of the lower uterine segment is called Hegars sign. Discoloration of mucous membranes of the vagina is called Chadwicks sign. Softening of the cervix is called Goodells sign. These options are incorrect as seen in rationale (A). 13. A patient at 9-weeks gestation is admitted to the emergency room complaining of a sharp pain in her right side with some vaginal spotting and nausea and vomiting. The nurse would expect: a. ectopic pregnancy b. abruptio placentae c. placenta previa d. threatening abortion Ans: A - a 9-week pregnant mother with sharp right-sided pain, some vaginal spotting, and nausea and vomiting may be experiencing entopic pregnancy. Abruption placenta will demonstrate enlarged abdomen, which is painful and rigid on palpation (and occurs much later in pregnancy). Signs and symptoms of placenta previa are painless uterine bleeding. The bright red blood may be inytermittent, occur in gushes, or more rarely, may be continuous. Abdominal examination reveals a soft (relaxed), nontender uterus. Signs and a symptoms of threatening abortion would be abdominal cramping. Uterine irritability or contractions, usually associated with minimal to moderate vaginal bleeding. 14. The first sign of preeclampsia usually is: a. hypertension b. epigastric pain c. pedal edema d. proteinuria Ans: A - the earliest sign of preeclampsia most commonly is hypertension, followed by proteinuria, and generalized edema. All of these signs and symptoms may not be present and may not present in this order. Epigastria pain is generally a late sign of preeclampsia and indicates liver involvement. Pedal edema is a normal occurrence in pregnancy. An increase in proteinuria indicates kidney involvement and generally follows hypertension. 15. Signs and symptoms of preeclampsia are: a. hyperreflexia, tachycardia, facial edema, and sudden weight loss b. fever, weight gain, fluid retention, and proteinuria c. tachycardia, hypergylcemia, and generalized d. hypertension., ptoteinuria, generalized edema, and weight gain Ans: D - signs and symtpoms of preeclampsia are hypertension, proteinuria, generalized edema, and weight gain. These options are incorrect as seen in rationale (D). 16. Rh incompatibility was once a leading cause of kernicterus and a significant cause of neonatal death. Incidence of Rh incompatibility has been greatly reduced by the administration of RhoD immune globulin (RhoGam) to: a. all Rh-negative mothers during pregnancy and within 72 hours of delivery or abortion of an Rh-positive infant or fetus b. all Rh-positive neonates immediately after birth c. all Rh-negative mothers within 72 hours after delivery d. all Rh-negative mothers every trimester and following delivery, if the fetus is Rh-positive Ans: A - the incidence of kerniterus has been greatly reduced by the administration of Rho (D) immune globulin (RhoGam) to all Rh-negative mothers during pregnancy and all unsentisized Rh-negative mothers within 72 hours after the delivery or abortion of Rh-positive newborns or fetuses. Kernicterus is a high level bilirubin in the blood, causing neurologic symtpoms. Rh incompatibility was a leading cause of kerniterus and a significant cause of neonatal death before RhoGam was developed. These options are incorrect as seen in rationale (A). 17. The nurse teaches patients that they should examine their breasts for lumps. a. One week after menstrual period b. One week before menstrual periods begins c. Each motion on the same date d. Two weeks after menstrual period Ans: A- the nurse teaches patient that all women over 20 years of age should examine their breasts for lumps 1 week after menstrual period. These options are incorrect as seen in rationale (B). 18. The following is considered essential to diagnose breast cancer:

a. ultrasonography b. core-needle biopsy c. mammography d. x-ray Ans: B- core-needle biopsy is considered essential to diagnose breast cancer. An ultrasonography mammography or x-rays are used to determine clinical manifestation of cancer of the breast but are considered essential to diagnose malignancy. These options are incorrect as seen in rationale (B). 19. Early clinical manifestations of breast cancer are: a. single painless, nontender, hard, Irregular, nonmobile mass b. multiple lumps, tender, soft movable mass c. one large nodule, tender, soft movable mass d. very small, painless, multiple, movable soft mass Ans: A- Early clinical manifestations of breast cancer are single painless, nontender, hard irregular nonmobile masses. Multiple lumps would not be an early sign of breast cancer but may be a late sign. The lump would not be a tender, soft movable mass. This may indicate a breast cyst. There may be one large nodule. This would be a late sign, but it would not be a tender, soft movable mass. Very small, painless, multiple, movable, soft masses may be cyst. 20. Women most at risk for breast cancer are: a. Over 65 years of age, married younger than 16 at first pregnancy b. Over 55 years, never married, with a family history of cancer c. Over 65 years of age, married, multiple sex partners at a young age d. Over40 years of age, never married, with a family history of lung cancer Ans: B- women over 55 years of age, who have never married, and who have a family history of cancer are the greatest risk for breast cancer. These options are incorrect as seen in rationale (B). 21. Radiation to the breast causes the skin to be: a. Dry, tender, red, swollen, and itchy b. Pale, moist, painful, and rash over area treated c. Cyanotic, dry, pigmented and painful d. Jaundiced, itchy, ulcerated and painful Ans: A-radiation will cause skin to become tender, red, swollen, and itchy. These options are incorrect as seen in rationale (A). The skin does not become pale, cyanotic, jaundiced, and painful. Ulcerated, or have a rash develop on treated area. 22. Side effects of chemotherapy are: a. Ptosis, diaphoresis, hypoxia, nausea and vomiting b. Photosensitivity, peripheral neuropathy and stomatitis c. Tremor, dizziness, diaphoresis and severe headaches d. Weight loss, irritability, diarrhea and ataxia, and severe headache Ans: B- side effects of chemotherapy are photosensitivity; peripheral neuropathy; symptomatitis; alopecia (hair loss); depressed red blood cells, white blood cells, and platelets; diarrhea, fatigue; menopausal symptoms; nausea; vomiting; peripheral nueropathy; sterility; and weight loss. These options are incorrect as seen in rationale (B). 23. A standard radical mastectomy involves the removal of: a. Cancerous mass and all normal tissue for clean margin b. The breast, axillary lymph nodes and pectoral muscles c. Axillary lymph nodes and pectoral muscles d. Resection of breast tissue and some skin from clavicle to costal margin Ans: B- a standard radical mastectomy involves the removal of the breast, axillary lymph nodes, and pectoral muscles. The whole breast, not just the cancerous mass, is removed in a standard radical mastectomy. Some normal tissue for a clean margin will be removed but not all-normal tissue. When you see all it is usually not the right answer. The removal of axillary lymph nodes, removal of breast, and overlying skin is a modified radical mastectomy. The difference between a modified radical and standard radical mastectomy is that, in the standard radical, the pectoral muscles are removed. The modified radical mastectomy is the most commonly performed procedure. Resection of breast tissue does not tell you how much breast tissue is being rejected or if it is cancerous or normal tissue. Skin would not be removed from the clavicle in any of the mastectomies. 24. Endometrosis is: a. abnormal location of endometrial tissue that is hormone dependent

b. abnormal growth of endometrial tissue within the uterus c. small endometrial abcessess with bleeding outside the uterine cavity causing severe pain d. excessive endometrial tissue in the uterus causing pain, bloating, and premenstrual tension Ans: A- endometriosis is the abnormal location of endometriosis tissue that is hormone dependent. Endometriosis is the extra uterine, endometriosis is progressive and may have widespread dissemination. The etiology is unknown. It has an inherited predisposition. It etiology is unknown. It has an inherited predisposition. it causes pelvic pain and infertility. It is not abnormal for endometrial tissue to grow in the uterus. This is normal. Endometriosis is not associated with abscesses or bleeding outside the uterine cavity. Excessive endometrial tissue in the uterus is not endmetriosis. 25. It is generally accepted that estrogen should not given to women: a. after a surgical procedure to enlarge the breast or intestinal disorders b. mastitis or when breastfeeding an infant c. with bone cancer, osteoporosis, or mastitis d. with known or suspected breast or uterine cancer Ans: D-estrogens should not be given to women with known or suspected breast or uterine cancer, or present thrombophlebitis, acute liver disease, cerebrovascular disease, or combined risks, such as obesity, varicosities, high blood pressure and heavy smoking. These options are incorrect as seen in rationale (D). 26. Treatment for endometriosis include: a. premain and dexamethasone b. pronestyl and dextran c. oral contraceptives and danocaine d. diazepam and miltown Ans: C-treatment for endometriosis includes oral contraceptives and Danocrine, a hormone that suppresses ovarian function. Premarin is hormone used to treat various estrogen deficiency stages. Dexamethasone is a glucocorticoid, long-acting, anti-inflammatory agent. Pronestyl is an antiarrhythmic used to treat a wide variety of ventricular and atrial arrhythmias. Dextrin is a volume expander-anticoagulant used in the emergency treatment of hypovolemic shock. Diazepam (Valium) is a sedative-hypnotic, anticonvulsant, and skeletal muscle relaxant used in the management of anxiety to provide preoperative sedation, light anesthesia, and to relax skeletal muscles. Miltown is a sedative hypnotic used as a sedative in the management of anxiety disorders. 27. Side effects of oral contraceptive are: a. pulmonary embolism, pulmonary edema, and hypertension b. anemia, mental depression, and hypotension c. syncope, diaphoresis, and hemorrhage d. iron loss, mental depression, and hypotension Ans: A- side effects of oral contraceptives are pulmonary embolism pulmonary edema, and hypertension. Anemia is a condition in which there is a reduction in the number of circulating red blood cells per cubic millimeter or the amount of hemoglobin per 100 mL; or the volume of packed red cells per 100 mL of blood. It exists when hemoglobin content is less than that required providing the oxygen demands of the body. Anemia may result from excessive blood loss, with use of chemotherapy), or in decreased blood cell formation, which is caused by drugs, ionizing radiation, deficiencies of vitamins (vitamin B12 deficiency, as in pernicious anemia), deficiency and folic acid. These options are incorrect as seen in rationale (A). 28. Oral contraceptives effects of: a. Prednisone, hydrocortisone, and medrol b. Anticoagulants, insulin, and antihypertensive agents c. Calcium gluconate, medrol, and dilantin d. Robitussin, dilaudid, and sudafed Ans: B- oral contraceptives decrease the effects of anticoagulants, insulin, and antihypertensive agents. Oral contraceptives are considered safest when given to nonsmoking women under 35 years of age that do not have a history of thromboembolism problems, diabetes mellitus, hypertension, or migraine headaches. Contraceptives do not decrease the effects of these drugs. Prednisone, hydrocortisone, and Medrol are steroids used to treat inflammations. Calcium gluconate is an electrolyte used in the treatment and prevention of calcium depletion in diseases associated with hypocalcemia (eg, hypoparathyroidism). Medrol is a steroid, and Dilantin is an anticonvulsant drug. Robitussin is used in the management of the cough associated with upper respiratory infections. Dilaudid, a n analgesic, is used in the treatment of moderate to severe pain. Sudafed is used in the symptomatic management of nasal congestion. Sudafed is a decongestant. 29. A woman in labor should be encouraged to void frequently so that

a. The descent of the presenting part is not hindered. b. Catheterization is not necessary c. Urine specimens are available to monitor labor. d. Rupture of the bladder does not occur. Ans: A- A full bladder will impede the descent of the presenting part because it fills the space through which the fetus needs to pass. It also significantly increases the amount of discomfort during contractions because the presenting part causes pressure on the distended bladder. 30. Which of the following factors in a mothers history would put a newborn at risk for infection? a. Rupture of membranes more than 24 hours before delivery. b. Bleeding episodes during the second trimester. c. Urinary tract infection during her third trimester. d. Use of Pitocin during labor. Ans: A- once the membranes ruptured, the infants protection from external organisms is compromised. When membranes have been ruptured more than 24 hours, the chance of infection is greatly increased.

General Nursing Board 101 Maternal and Child Nursing Pre-Test 2 http://nsgbrd101.proboards.com/index.cgi?board=answers&action=display&thread=9 Maternal and Child Nursing Pre-Test 2 Post by admin on Aug 18, 2010, 1:07pm 1. A neonates failure to pass meconium within the first 24 hours after birth could indicate: a. Intussusception b. Dehydration c. Celiac disease d. Hirschprungs disease Ans: D - a newborns failure to pass meconium within the first 24 h after birth may indicate Hirschsprungs disease, a congenital disorder of the large intestine. It is characterized by absence or marked reduction of parasympathetic ganglion cells in the intestinal wall. This impairs the intestinal motility and causes severe constipation. Without prompt treatment, a neonate will develop bowel obstruction and may die within 24 h. Hirschsprungs disease is believed to be a familial, congenital defect, occurring more often in males than in females. Clinical manifestations usually appear shortly after birth, but mild symptoms may not be recognized until later in childhood or during adolescence. The newborn with Hirschsprungs disease commonly fails to pass meconium within 24 to 48 h, shows signs of bowel obstruction, abdominal distention, irritability, poor sucking reflex, refusal to take feedings, failure to thrive, dehydration, and liquid stools. Surgical treatment is necessary, but is delayed until the infant is at least 10 months old. Management until surgery consists of daily colonic lavages to empty the bowel. If total obstruction is present in the newborn, a temporary colostomy or ileostomy is necessary to decompress the bowel. Intussusception is when the bowel turns back into itself (telescoping). Intussusception is most common in infants and occurs three times more often in males than in females. Signs and symptoms of intussusception are intermittent attacks of colicky pain; vomiting of stomach content, currant jelly stools, which contain a mixture of blood and mucus; tender distended abdomen, with a palpable, sausage-shaped abdominal mass. Dehydration will not cause a failure to pass meconium. Celiac disease is characterized by poor food absorption and intolerance for gluten, a protein in wheat and wheat products. With treatment such as eliminating gluten from the clients diet, prognosis is good. The cause is unknown, but females are affected more often than males. This disorder produces the following clinical manifestations; recurrent attacks of diarrhea, steatorrhea (fat in stool), abdominal distention, stomach without cramps, weakness, anorexia, and increased appetite without weight gain. Symptoms develop during the first year of life when gluten is introduced in the childs diet as cereal. Treatment requires permanent elimination or reduction of gluten from the clients diet. Supportive treatment may include supplemental iron, vitamin B12, folic acid, reversal of electrolyte imbalances (by IV infusion, if necessary), corticosteroids (prednisone or hydrocortisone), and vitamin K for hypoprothrombinemia. 2. Three-week-old Billy is admitted to the hospital with a history of spitting up since he was 1 week old. His mother says, Billys spitting has now increased to what I would call forceful vomiting. The most likely diagnosis for Billy would be: a. Hirshprungs disease

b. Celiac disease c. Pyloric stenosis d. Intussusception Ans: C-when a 3-week-old baby continues to spit up large amounts of formula and then forceful vomiting, pyloric stenosis may be indicated. Obstruction of the sphincter is one of the most common surgical disorders of early infancy. This disorder is usually seen soon after birth, with vomiting becoming progressively more severe and projectile. It is five times more common in male infants than in female infants. The cause of pyloric stenosis is unknown. Diagnosis is made by upper GI x-ray studies. 3. A 10-year-old female client is diagnosed with insulin-dependent diabetes mellitus. She asks the nurse why she cannot take a pill her Aunt Jessie does, rather than insulin shots. The most correct response by the nurse would be: a. The pill stimulates cells in the pancreas to produce insulin, and your cells are not able to produce insulin b. The pills will stimulate insulin production in the adult pancreas but will not in the childs pancreas c. When you are able to cut down on the amount of sweets you eat, then maybe you can start taking the pill d. Your doctor, makes that decision, ask him Ans: A- when a child has insulin dependent diabetes mellitus (IDDM), the beta cells of the pancreas can no longer produce insulin. The patient must like insulin by subcutaneous (SC) injection, and will have to take insulin for the rest of his or her life. A patient with noninsulin dependent diabetes mellitus (NIDDM) will have some active beta cells; an oral hypoglycemic medication will stimulate these beta cells to produce insulin. The statements, The pills will stimulate insulin production in the adult pancreas but will not in child, is an incorrect statement. Oral hypoglycenmic medication will stimulate live beta cells to produce insulin in a childs pancreas or an adult pancreas in a child. These option are incorrect as seen in rationale (A). 4. A 2-year-old female client is brought to the emergency room after waking up with a bark-like cough and stridor. On arrival to the ER, she has respiratory distress and is a febrile. The diagnosis is croup. The nurse instructs the parents to: a. Perform percussion and postural drainage before putting the patient to bed and before meals b. Encourage frequent coughing and deep breathing c. Run a cool mist vaporizer in patients room during the day d. Follow a schedule of postural drainage and increase fluid intake Ans: C- the nurse should instruct the parents of a child with croup to run a cool mist vaporizer in the childs room at night and during the day, and take the child into the shower with them in an acute care situation. Croup is a severe inflammation and obstruction of the upper airway, occurring as laryngotracheobroinchitis (most common), laryngitis, and acute spasmodic laryngitis. Croup is a childhood disease-affecting males more often than females (typically between 3 months and 3 years), usually during the winter months. Croup usually results from a viral infection, but can be caused by bacteria. Most children are sued at home with rest, cool humidification during sleep, and antipyretics (such as acetaminophen) to relieve symptoms. In an acute case, taking the child into a shower will provide the child with a more humid atmosphere. These options are incorrect as seen in rationale (C). 5. The symptoms most commonly seen in croup are: a. Wheezing, colicky pain and vomiting b. Stridor, rapid pulse and bark-like cough c. Drooling, rapid pulse and occasional hoarse cry d. Fever, vomiting and abdominal retractions on inspiration Ans: B-the symptoms most commonly as seen in croup are stridor (labored breathing with retractions), rapid pulse, and a bark-like cough. These options are incorrect as seen in rationale (B). 6. Which of the following is the most threatening to a hospitalized toddlers autonomy? a. Frequent visits by parents and friends of the family b. Participation in playroom activities with other children c. Complete bed rest d. Riding to the x-ray department in a wheelchair in a hospital gown Ans: C- one of the most threatening things to a hospitalized toddlers autonomy is complete bedrest. The toddler is just beginning to assert independence and is very active, and does not want to be kept in bed. Bedrest is very threatening to a toddler who does not understand the reason for it. Frequent visits may parents and friends help the child feel safe and not abandoned. This is not threat to the childs autonomy. A

toddler loves to play with other children, even if they do not known them. This encourages autonomy (not threatens it); normally, toddlers make friends easily. Riding to x-ray in a wheelchair could be fun for a child, not a threat to autonomy. 7. Which one of the following phrases most accurately describes myelomeningocele? a. The incomplete fusion of the vertebrae at one level that may have an overlying dimple or tuft of hair b. Herniation of a portion of the spinal cord and meninges into a cyst c. The incomplete fusion of one or more of the vertebral laminae d. A cyst formation containing CSF, blood and meninges Ans: B- myelomenigocele is the most severe neural tube defect (NTD), involving protruding sac-like structure tat contains meninges, spinal fluid, and neural tissue. The spinal nerve roots, may terminate in the sac, significantly affecting motor and sensory function below that point. The incomplete fusion of the vertebrae at one level with An overlying dimple of turf protrusion of the spinal tissue is called spina bifida cystica. This type of defect is usually in the lumbosacral area. There are two classifications of spinal bifida cystica; myelomeningocele and meningocele. Spina bifida cystica is a general classification for these disorders; therefore, this is an incorrect option. Meningocele is the formation of a sac-like cyst. Which contains meninges and spinal fluid that produces through a defect in the bony spine. 8. When explaining how to correctly collect specimen for identification of pin worms, the nurse should tell the mother to: a. Bring in a fresh stool specimen b. Administer a laxative the night prior to collection of a specimen c. Place a tongue blade covered with tape over the childs anus d. Have the child defecate and then smear a small amount of stool on a slide Ans: C- to collect a specimen for the identification of pin worms, a tongue blade covered with tape is placed over the childs anus during the night for a specimen. The sticky side of the tape is placed on the rectum. During the right, the worms crawl into the anus and lay their eggs. The eggs will stick to the tape on the tongue blade. A fresh stool specimen may not have any pin worms in it, but the child may still have pin worms. Fresh specimens are best for revealing parasites or larvae; therefore, a collected specimen should be taken directly to the laboratory for examination. These options are incorrect as seen in rationale (C) and (A). 9. Activated charcoal is administered to the child who has ingested a poison substance in order to: a. Induce vomiting b. Increase the effectiveness of ipecac c. Increase movement through the GI tract d. Absorb the compound Ans: D- activated charcoal is administered to the child who has ingested a poisonous substance to absorb the compound. Activated charcoal effectively absorbs most poisons, with the exception of cyanide. It also absorbs ipecac syrup; therefore, the emetic should be given and be allowed to exert its effect before the charcoal is given. Charcoal, however, is most effective if administered within 30 min of ingesting the poison. These options are incorrect as seen in rationale (D). 10. When giving parents anticipatory guidance about accident prevention for toddlers, the nurse tells the parents: a. If the toddler feels the heat, he learns that a stove is hot b. Falls are not as great a danger now as they were during infancy c. Areas previously childproofed may now be accessible to toddler d. Toddlers understand the word NO and will listen to parental rules Ans: C- when giving parents anticipatory guidance for toddlers, the nurse tells the parents that areas previously child-proofed may now be accessible to the toddler. Anticipatory guidance is the ideal way to handle a problem. Prevent it deal with it before it becomes a problem. Parents know what to expect will be prepared for a problem when it appears. The statement , If the toddler feels the heat, she learns that the stive is hot, is incorrect. The child does not have to experience every danger to prevent it from happening. If a parent waits for a child to be burned, the burn could be fatal. The statement, Falls are not as great a danger now as during infancy, is not true. Falls continue to be a great danger throughout life. The statement, Toddlers understand the word no and will listen to parental rules, is a false statement. Toddlers tend to ignore the word no and find it very difficult to obey their parents. They are interested in

establishing their autonomy. 11. A mother brings her 3-year-old son and her 8-month-old daughter to the health center. She states that her son enjoys pouring and playing with water. The water safety rule for the bathroom is reviewed with the mother. The only correct rule given here is: a. Never leave children alone in the bathtub b. Never leave a child in the bathtub unless an older child is in with them c. Have the child test the water temperature with his hand d. It is safe to leave children alone in bathtub if there is one 12 inches of water Ans: A- a water safety rule given to parents should be to NEVER leave children alone in the bathtub. A 3year-old child is to young to take care of any child in bathtub, this is not a safety rule. The adults responsibility is to check the temperature of there is only 12 inches of water; they can turn on the faucet and fill the tub or drown in 12 inches of water. Note: The safety rules are many but the only one given in this situation is option (A). 12. The nursing diagnosis appropriate for health promotion of a well toddler is: a. Potential for injury related to increased mobility b. Activity intolerance related to rapid growth c. Alteration in growth and development related to rapid growth d. Alteration in bowel elimination related to interest in playing, not drinking Ans: A- the nursing diagnosis appropriate to health promotion of the well toddler is potential for injury related to increased mobility. A well toddler does not intolerance to activity; they love activity. This nursing diagnosis is incorrect. A well toddler does not have an alteration in growth and development. This nursing diagnosis is incorrect. A well toddler normally does not have an alteration in bowel elimination. This nursing diagnosis is incorrect. 13. A mother brings her 2-year-old son to the clinic for his measles, mumps and rubella vaccine (MMR). Prior to giving the vaccine, it is important for the nurse to ask if he has: a. Recently been checked for anemia b. Ever experienced an allergic reaction to eggs c. Been in close contact with anyone with rubella (German measles) d. Been in close contact with a pregnant relative or baby sitter Ans: B- prior to giving a child a vaccine for measles, mumps, and rebulla (MMR), the nurse should ask the parents if the child is allergic to eggs because they vaccinated for measles, mumps, and rebulla. If the child has been in contact with someone who had rebulla (German measles), he or she can still get his or her MMR vaccine. Being exposed to measles would not affect the MMR vaccine given to the child unless the child was exposed 10 to 14 days before scheduled for their first MMR vaccine. Then they would be given gamma and MMR vaccine at a later date. The child being in contact with a pregnant relative or babysitter has no effect on the MMR vaccine given to the child. If the child has rebulla (German measles), should not be around a pregnant woman. If the pregnant, the chance of her child having a birth defect are high. Congenital rubella is by far the most serious form of the disease. Intrauterine rebulla infection, especially during the first trimester, can lead to spontaneous abortion or stillbirth, as well as single and multiple birth defects. As a rule, the earlier the infection occurs during pregnancy, the greater the damage to the fetus. 14. In explaining the use of time out as a form of discipline for a 2-year-old child, the nurse tells the parents: a. To send the child to his or her bedroom for 20 minutes b. To allow the child to take his or her own favorite toy to the room c. The total time out should be no more than 2 to 3 minutes d. To explain to the child why the behavior was bad after( time out) Ans: C-in explaining the use of time out as a form of discipline for a 2-year-old child t he nurse tells the parents that the total time out should be no more than 2 to 3 min. a 2-year-old child will forget why they have no remain in a room after a few minutes and will become upset if they are left in their room. This option is incorrect seen in rationale (c). allowing a child a favorite toy in time out is not disciplinary action; the child may believe they are put in time out just play for a little while. The parent needs to explain to the child before time out why their behavior was not acceptable. A child should not be t old that what they did or did not do is bad because the child may then think of themselves as bad.

15. The behavior that would indicate to parents that a toddler is ready to begin toilet training is: a. Showing interest in flushing the toilet b. Staying dry throughout the night c. Spending time in the bathroom with parents d. Pulling a wet diapers or taking off wet diapers Ans: D- the behavior that would indicate to parents that a toddler is ready to begin toilet training is when the toddler pulls off water diapers. The child is uncomfortable wearing a wet diaper and can recognize that they are the one who is wetting them. The parent can tell them, If you do not like your diaper wet, then you can g (pee) in the toilet. The child may pull off t he wet diaper and ask the mother to take them to the toilet. Flushing the toilet indicates that the toddler likes to hear the water run and has nothing to do with being ready to be toilet trained. Many children stay dry throughout the night long before they are ready to be toilet trained. If they do not their diapers at night, it is probably because they did not drink much fluid before going to bed. Spending time in the bathroom with parents does not indicate that they are ready to begin toilet training. They may just want to be near their parents. 16. A mother of an 18-month-old child tells the nurse that her child is a finicky eater. The best advice that the nurse can give to the mother is: a. Give small amounts and cut food into small pieces allow for finger feeding b. Dont force the child to eat but make sure that the child drinks all of his milk c. Serve small portions and make the child sit at the table until he eats all of his foods d. Serve the child in front of the TV to keep his attention Ans: A- when an 18-month-old toddler is a finicky eater the best advice the nurse can give the mother is, Give small amounts and cut food into small pieces that follow for finger feeding. A toddler is interested in autonomy; allowing the child to feed himself gives the child the autonomy that he is seeking. Children never be forced to eat all of their food or drink all of I their mil because they might aspirate the food or the milk. This method will not encourage them to eat the food. Making a toddler sit at the table until eat all of their food will make the child hate eating. The child may be at the tale for an hour and still not have eaten all of his food. This is punishment that is associated with eating, and certainly will not encourage the child to eat. If a toddler who is finicky eater is served food in front of the TV, the toddler will watch TV and not eat. 17. When toddler says, NO to eating a food, they may be: a. Saying they do not like the food, be testing their parents, be practicing independence b. Deciding the types of foods they like and dislike c. Trying to impress on their parents that they will have the right to choose their own food d. Saying no because they like the word; it has no meaning at all and is just a habit that children develop Ans: A- when a toddler says no to eating a food, they may not like the food, may be testing the parents, or be practicing independence. Toddlers know what foods they like and dislike, but this is not always the reason they say no eating a food. Toddlers do not have the cognition to think abut impressing people. Toddlers say no for many reasons; option (A) covers some of them. 18. When a doctor has a consent form signed for a surgical procedure on a child, the nurse knows that: a. Only the parent or legal guardian can sign the consent form b. The person giving consent must be at least 18 years old c. The risk and benefits of procedures are parts of the consent process d. A mental age of 7-year-old or older is required for a consent to be considered informed Ans: A- when the doctor is having a consent form signed for a surgical procedure on a child, the nurse knows that only a parent or legal guardian can sign the consent for. These options are incorrect as seen in rationale (A). 19. The nurse is planning how to prepare a 4-year-old child for a diagnostic procedure. Guidelines for preparing this preschooler should include: a. A plan for a short teaching session of about 35 minutes each day b. Telling the child that the procedure is not a form of punishment c. Keeping equipment out of the childs view d. Telling the child that the procedure will be simple Ans: B-the nurse knows that the guidance for preparing a preschooler for diagnostic procedure is to tell the child that the is not a form of punishment. Preschool children believe that diagnosis procedures and treatments are punishments for something they have done. This answer calls for a short teaching session;

35 min is not a sort teaching session. A preschool child will lose interest in a teaching session that last more than a few minutes; dolls or stuffed animals must be used to explain procedures and keep the childs attention. The child needs to see the equipments that will be used for diagnostic purposes for treatment the day before so that they will be familiar and not be afraid of it. The nurse should not tell the child that the procedure will be simple because the procedure may not be simple to the child. The child will be more afraid when the procedure hurts or causes them some discomfort. The child will not trust the nurse and will be more fearful of the hospitals treatments and it personnel. 20. The nurse is preparing a 6-year-old child before obtaining a blood specimen by venipuncture. The child tells the nurse that he does not want to lose his blood. The appropriate approach by the nurse would be to: a. Explain that the process will not be painful and that it will be over quickly b. Discuss with the child how the body is always in the process of making blood c. Suggest to the child that he does not have to worry about losing a little bit pf blood because he is old enough now not to be afraid d. Tell the child that he will not need a Band-Aid after ward because it is such a simple procedure. If he does not watch the nurse draw the blood, it will not hurt Ans: B- when a nurse is preparing a child for a venipuncture, the nurse needs to understand that 6-year-old child has a great fear of bodily injury. Because children heave an inadequate comprehension of medical events, they have a fear of invasive and /or painful procedures. The nurse should discuss with the child how the body is always in the process of making blood. Discussing the procedure with the child and allowing them to express their feelings will help to reduce their anxiety. If the nurse tells the child that the procedure is not painful and they will not even need a Band-Aid, the nurse is not being honest with the child. When the child then experience pain, he or she will no longer trust nurses and will fear every procedure. A nurse should never tell a child or an adult not to worry; this does not help them cope with their fears and it makes the patient feel as if the nurse is not concerned. 21. When administering a gavage feeding or medication to an infant through a nasogastric tube, the nurse should: a. Lubricate the tip of the feeding tube with petroleum to facilitate passage b. Check the placement of the tube by inserting 20 ml of sterile water c. Administer feedings over a period of 15 to 30 minutes d. Place the infant on its right side or abdomen for 1 hour after feeding Ans: D-when administering a gavage feeding or medication through a nasogastric (NG) tube to an infant, the nurse should place the infant on the right side of abdomen for 1 h after the feeding. The stomach is on left side; therefore, the infant should not be laced on the left side as the pressure on the stomach can cause the infant to vomit. If the infant is placed on the right side or on the abdomen and vomits, the vomit come out of the mouth and not be aspirated. Being in this position for 1 h will allow time for the food or medication to leave the stomach. The questions is when administering a gavage feeding. Not when putting an NG tube into the stomach. Read your stem carefully. The placement of the NG tube should not be done with water, because the tube might be in the lungs. The correct procedure for checking NG placement is to push air through the NG tubing with a large syringe while listening over the stomach with a stethoscope. If a bubbling sound (air) is heard, then the nurse knows the NG tube in the stomach, or withdrawn stomach content and check acidity. There is no reason to administer tube feedings over a period of 15 to 30 min. there is an order for continuous tube feeding, then the tube feeding is continuous not over 15 to 30 min. 22. In preparing to give enemas until clear results (colonic lavage) to an 18-month-old child before surgery for Hirschprungs disease, the nurse would anticipate that the physician would order: a. Tap water (120 ml) b. Fleet solution (200 ml) c. Saline (300 ml) d. Oil retention (300 ml) Ans: C- when giving enemas until clear to an 18-mnoth-old child surgery for a megacolon (Hirschsprungs disease), the nurse would anticipate that the physician would order 300 mL saline enema. Hirchsprungs disease is a congenital disorder of the large intestine, characterized by the absence or marked reduction of parasympathetic ganglion cells. The disorder impairs intestinal motility and causes severe constipation. Before surgery, at least once a day, a large amount of saline is given as an enema (colonic lavage). Enemas given with the standard amount of fluid are not sufficient because the colon is so large and laxatives will not clear the colon adequately. Saline (salt) is used to draw as much fluid as possible into the intestines. These options are incorrect as seen in rationale (C).

23. In relation to growth, the parents of a toddler can expect: a. A growth spurt at about 18 months of age b. A 4 to 6 lbs gain of weight between ages 3 and 4 c. Small weight losses and larger gains related to changing appetite d. Little increase in height before age 3 years Ans: B- in relation to growth, the parents of a toddler can expect a 4-6 pound gain in weight between ages 3 and 4. these options are incorrect as seen in rationale (B). 24. An 18-month-old child is seen in the clinic for a well-child check-up. While completing the assessment, the nurse notes a protruding abdomen. The nurses assessment is that: a. The child may be constipated b. Further evaluation is warranted c. An abdominal tumor must be ruled out d. This is a normal finding Ans: D- an 18-month-old child seen in a clinic for a well child assessment will have a protruding abdomen, normal for this age. The abdominal muscles are not yet strong enough to keep the abdomen pulled in, but this will change as the child grows older. These options are incorrect as seen in rationale (D). 25. A premature infants dietary intake needs to be particularly sufficient in: a. Glucose b. Iron c. Zinc d. Vitamin C Ans: B- maternal iron stores are adequate for the first 4 to 5 months of age in the full-term infant, but are reduced considerably in premature infants of multiple births. When exogenous sources of iron are not supplied to meet the infants growth demands following depletion of fetal iron store, iron deficiency anemia results. Iron is necessary for hemoglobin synthesis. Hemoglobin is the oxygen carrying molecule on he red blood cells. These options are incorrect as seen in rationale (B). 26. Visual development in an 18-month-old is directly related to parental need for anticipatory guidance related to: a. Prevention of falls b. Selection of a dentist c. Signs of infections and need for glasses d. High dietary intake of vitamin A Ans: A- visual development in toddler (18-month0old) is directly related to parental need for anticipatory guidance related to prevention of falls, because the childs depth perception is not fully developed. These options are incorrect as seen in rationale (A). 27. Toddlers are most interested in play that involves: a. Watching TV, especially cartoons and playing Nintendo b. Interaction with other toddlers and playing games with them c. Active use of small and large muscle groups for climbing and running d. Adults telling or reading stories for 30 minutes at a time Ans: C- toddlers are most interested in play that involves the active use of small and large muscle groups for climbing and running. At this age, the toddler is very active and very curious. It is dangerous age because toddlers can climb and get into things. A toddler may enjoy cartoons but will not sit still for very long. The toddler is not old enough to play Nintendo. Toddlers are not interested in playing games with other children. They like to sit beside each other and pay, but not share. Toddlers enjoy having adults read to them, but only for 3 to 5 min at a time; their attention span is very short. 28. The mother of a 2-year-old boy tells the nurse that the child insists on taking a favorite blanket to bed with him at night and at naptime. The mother asks the nurse if she should send the blanket to nursery school with him. The best response by the nurse is: a. No, this is a good time for him to give it up and grow up b. No, he doesnt need it away from home; he will be too busy playing c. Yes, it will help him feel secure in his new place d. Yes, it may cause severe emotional problems to take it away now Ans: C- if the mother tells the nurse that her 2-year-old son insists on taking a favorite blanket to nursery

school with him and wants to know if she should allow this, the nurses best response is, Yes, it will help him feel secure in his new place. Telling a toddler he needs to grow up and removing his security blanket will make the child insecure and afraid. Telling the mother that the child does not need the blanket away from home and that he will be too busy playing to want his blanket is not true. May toddlers have blankets or toys they keep with and carry around all of the time because the objects offers them security. Taking this security away when they are away from home would cause the child to feel insecure and cause emotional upset. To tell a mother that it would cause severe emotional problems if he blanket was taken away is not true; it may serious emotional problems. If something happened to the blanket (it might get lost), then the mother may believe that the child with have been severe emotional problems. 29. Sigmund Freud believed that the major task of a toddler period is: a. Anxiety over separation from mother b. Toilet training c. Development of the superego d. Developing gender identity Ans: B Sigmund Freud believed that the major task of the toddler period is toilet training, and he described this stage as the genital stage. Bowel training may be accomplished between ages 18 and 24 months. Bladder training may not be accomplished until ages 3 to 5 years especially night-time control. Eriksons phase 1 is concerned with acquiring a sense of basic trust while overcoming a sense of mistrust. The trust acquired in infancy is the foundation for all the succeeding phases. Separation anxiety is prominent during the latter half of infancy (8 months) and is still present to some degree when toddlers are 1 to 3 years old, and is even seen in preschool children. Toddlers are more willing to meet strangers and will tolerate longer periods of separation. There is less of the extreme fear of separation that was prominent during the latter half of infancy. Development of superego, or conscience has its beginning toward the end of the toddler years (age 3) and is a major task for the 3 to 5-year-old preschool-age child. Learning right from wrong and good from bad is the beginning of morality. Most children are aware of their gender and the expected set of related behaviors by 1 to 2 years of age. This is called developing gender identity. Although toddlers might be aware of their particular sex, they do not possess the language and cognitive skills to investigate sexual identity as fully as preschool age children. Freud has long recognized this task by describing this period as the genital stage. 30. A mother tells the nurse that at 2 years of age, her daughter is speaking new words almost daily. For the past month, the child has shown little interest in new vocabulary. The nurse should: a. Explain that the childs energies now are focused on developing motor skills b. Suggest that the mother spend an hour reading to the toddler daily c. Evaluate the possibility of hearing loss and refer for testing d. Perform a Denver Developmental Screening Test to check for delays in vocabulary development Ans: A a mother tells the nurse that at 2 years of age, her daughter was speaking new words almost daily. For the past month, however, she shows little interest in new vocabulary. The nurse should explain that the childs energies are now focused on developing motor skills.

General Nursing Board 101 Maternal and Child Nursing Comprehensive http://nsgbrd101.proboards.com/index.cgi?board=answers&action=display&thread=10 Maternal and Child Nursing Comprehensive Post by admin on Aug 18, 2010, 1:16pm 1. A client in the 28th week gestation comes to the emergency department because she thinks that she is in labor. To confirm a diagnosis of preterm labor, the nurse would expect physical examination to reveal: a. Irregular uterine contractions with no cervical dilatation b. Painful contractions with no cervical dilatation c. Regular uterine contractions with cervical dilatation d. Regular uterine contractions with no cervical dilatation Ans: C regular uterine contractions (every 10 minutes or more) along with cervical dilation before 36 weeks gestation or rupture of fluids indicates preterm labor. Uterine contractions without cervical change dont indicate preterm labor. 2. A client in the active phase of labor has reactive fetal monitor strip and has been encouraged to walk.

When she returns to bed for a monitor check, she complains of an urge to push. When performing vaginal examination, the nurse accidentally ruptures the amniotic membranes, the umbilical cord comes out. What should be done next? a. Put the client in a knee-chest position b. Call the physician or midwife c. Push down on the uterine fundus d. Set up for a fetal blood sampling to assess for fetal acidosis Ans: A the kneetochest position gets the weight off the baby and umbilical cord, which would prevent blood flow. Calling d physician or midwife and setting up for blood sampling is important, but they have a lower priority than getting d baby off the cord. Pushing down on d fundus would increase d danger by further compromising blood flow. 3. A client is attempting to deliver vaginally despite the fact that her previous delivery was by cesarean section. Her contractions are 2 to 3 minutes apart, lasting from 75 to 100 seconds. Suddenly, the client complaints of intense abdominal pain and the fetal monitor stops picking up contractions. The nurse recognizes that which of the following has occurred? a. Abruptio placentae b. Prolapsed cord c. Partial placenta previa d. Complete uterine rupture Ans: D in complete uterine rupture, the client would feel a sharp pain in the lower abdomen and contractions would cease. Fetal heart rate would also cease within a few minutes. Uterine irritability would continue to be indicated by the fetal heart monitor tracing with abruption placentae. With a prolapsed cord, contractions would continue and there would be no pain from d prolapse itself. There would be vaginal bleeding with a partial placenta previa, but no pain outside of the expected pain of contractions. 4. A client with gravida 3 para 2 at 40 weeks gestation is admitted with spontaneous contractions. The physician performs an amniotomy to augment her labor. The priority nursing action is to: a. Explain the rationale for the amniotomy to the client b. Assess fetal heart tones after the amniotomy c. Ambulate the client to strengthen the contraction pattern d. Position the client in a lithotomy position to administer perineal care Ans: B - the nurse should assess fetal heart tones. After an amniotomy is performed, the umbilical cord may be washed down below the presenting part and cause umbilical cord compression, which would be indicated by variable deceleration on the fetal heart tracing. An explanation of the rationale for amniotomy would be given before d procedure. After assessing the fetal response to the amniotomy, perineal care s provided. The nurse would ambulate client only if the presenting part were engaged. 5. The nurse can consider the fetuss head to be engaged when: a. The presenting part moves through the pelvis b. The fetal head rotates to pass through the ischial spines c. The fetal head extends as it passes under the symphysis pubis d. The biparietal diameter passes the pelvic inlet Ans: D d fetuss head s considered engaged when the biparietal diameter passes d pelvic inlet. The presenting part moving through d pelvis s called descent. The head flexing so that the chin moves closer to d chest s called flexion. Rotation of the head to pass through the ischial spines is called internal rotation. Extension of the head as it passes under d symphysis pubis s called extension. 6. A client is experiencing true labor when her contraction pattern shows: a. Occasional irregular contractions b. Irregular contractions that increase in intensity c. Regular contractions that remain the same d. Regular contractions that increase in frequency and duration Ans: D- regular contractions that increase in frequency and duration as well as intensity indicate true labor. The other choices dont describe d contraction pattern of true labor. 7. A client is admitted to the hospital with contractions that are about 1 to 2 minutes apart and reveal that her cervix is dilated 8 cm. The client is in which stage of labor? a. Latent phase b. Active phase c. Third stage d. Transitional phase

Ans: D- d client is in d transitional phase of labor. This phase of labor is characterized by cervical dilation of 8 to 10 cm and contractions that are about 1 to 2 minutes apart and last for 60 to 90 seconds with strong intensity. In the latent phase, the cervix is dilated 0 to 3 cm and contractions are irregular. During the active phase, the cervix is dilated to 4 to 7 cm and contractions are about 5 to 8 minutes apart and last 45 to 60 seconds with moderate to strong intensity. The 3rd stage of labor extends from delivery of the neonate to expulsion of the placenta and lasts from 5 to 30 minutes. 8. A client in the second stage of labor experiences rupture of membranes. The most appropriate intervention by the nurse is to: a. Assess the clients vital signs immediately b. Observe for prolapsed cord and monitor fetal heart rate c. Administer oxygen through a face mask at 6-10 L per min d. Position the client on her side Ans: B the nurse should immediately check for prolapsed cord and monitor FHR. When the membranes rupture, the cord may become compressed between the fetus and maternal cervix or pelvis, thus compromising fetoplacental perfusion. It isnt necessary to position the client on her left side, monitor maternal vital signs, or administer oxygen when the clients membrane rupture. 9. A client in labor is being monitored by an internal electronic device to evaluate fetal station. The nurse measures the duration of her contractions by: a. Measuring from the beginning of the increment to the end of the decrement b. Measuring from the beginning of one contraction to the beginning of the next c. Measuring from the beginning of the decrement to the end of the increment d. Using an intrauterine catheter that measures increases in contraction Ans: A- the duration of a contraction is measured from the beginning of the increment to the end of the decrement. Measuring from the beginning of one contraction to the beginning of the next reveals frequency. Measuring from the beginning of one contraction to the beginning of the next reveals frequency. Measuring during the acme phase of a contraction reveals intensity (measured with an intrauterine catheter or by palpation). 10. A client is receiving magnesium sulfate to help suppress preterm labor. The nurse should watch for which sign of magnesium toxicity? a. Headache b. Loss of deep tendon reflexes c. Palpitations d. Dyspepsia Ans: B magnesium toxicity causes signs of central nervous system depression, such as loss of deep tendon reflexes, paralysis, respiratory depression, drowsiness, lethargy, blurred vision, slurred speech, and confusion. Headache may be an adverse effect of calcium channel blockers, which are sometimes used to treat preterm labor. Palpitations are an adverse effect of terbutaline and ritodrine, which are also used to treat preterm labor. Dyspepsin may occur as an adverse effect of indomethacin, a prostaglandin synthesize inhibitor, used to suppress preterm labor. 11. When assessing a postpartum client for uterine bleeding, the nurse finds the fundus to be boggy. After fundal massage, the physician prescribes 0.2 mg of methylergonovine (Methergine) by mouth. What should the nurse tell the client? a. Methergine is commonly used to help the uterus contract so that the bleeding will decrease. You may experience more cramping as your uterus becomes firmer. b. You will probably take this medication until you are discharged from the hospital. Every patient usually needs to take this medication. c. If your blood pressure is low, you wont be able to take this medication; I will establish a new IV line so I can start Pitocin again. d. Most people dont experience additional pain or cramping from taking this medication. Ans: A Methylergonovine, an ergot alkaloid, is commonly given to stimulate sustained uterine contraction. It allows the uterus to remain contracted and firm, thus decreasing postpartum bleeding. Abdominal cramping, which may become painful, is a common adverse effect. Methergine is discontinued when the lochia flow has decreased or the client complains of severe cramping. Clients may need only a few doses of Methergine to keep the uterus contracted. Taking Methergine is contraindicated in clients with hypertension.

12. The nurse is providing care for a postpartum client. Which of the following conditions would place this client at greater risk for postpartum hemorrhage? a. Hypertension b. Uterine infection c. Placenta previa d. Severe pain Ans: C d client with placenta previa is at greatest risk for postpartum hemorrhage. In placenta previa, the lower uterine segment doesnt contract as well as the fundal part of the uterus; therefore, more bleeding occurs. Hypertension, severe pain, and uterine infection dont place the client at increased risk for postpartum hemorrhage. 13. A client has delivered twins. What is the most important intervention for the nurse to perform? a. Assess fundal tone and lochia flow b. Apply a cold pack to the perineal area c. Administer analgesics as ordered d. Encourage voiding by offering the bedpan Ans: A women who experience a twin delivery are at a higher risk for postpartum hemorrhage due to overdistention of d uterus, which causes uterine atony. Assessing fundal tone and lochia flow helps to determine risks for hemorrhage. Applying cold packs to d perineum, administering analgesics as ordered, and offering d bedpan r all significant nursing interventions, however, detecting and preventing postpartum hemorrhage s most important. 14. Which of the following is a normal physiological response in the early postpartum period? a. Urinary urgency and dysuria b. Rapid diuresis c. Decrease in blood pressure d. Increased motility of the GI system Ans: B in d early postpartum period there s an increase in the glomerular filtration rate and a drop in progesterone levels, which result in rapid diuresis. There should be no urinary urgency, although a woman may be anxious about voiding. There is minimal change n blood pressure following childbirth and a residual decrease in gastrointestinal motility. 15. During the 3rd postpartum day, which of the following would the nurse be most likely to find in the client? a. Shes interested in learning more about newborn care b. She talks a lot about her birth experience c. She sleeps whenever the baby isnt present d. She requests help in choosing a name for the baby Ans: A d 3rd to 10th days of postpartum care are the takinghold phase, in which the new mother strives for independence and s eager for her baby. B, C & D describe d phase n which d mother relives her birth experience. 16. Which of the following circumstances is most likely to cause uterine atony, leading to postpartum hemorrhage? a. Hypertension b. Cervical and vaginal tears c. Urine retention d. Endometriosis Ans: C urine retention is most likely to cause uterine atony and subsequent postpartum hemorrhage. Urine retention causes a distended bladder to displace the uterus above the umbilicus and to the side, which prevents d uterus from contracting. The uterus needs to remain contacted if bleeding is to stay within normal limits. Cervical and vaginal tears can cause postpartum hemorrhage, but in the postpartum period, a full bladder is the most common cause of uterine bleeding. Endometritis, an infection of the inner lining of the endometrium, and maternal hypertension dont cause postpartum hemorrhage. 17. When assessing a clients episiotomy, the nurse should be especially careful to observe:

a. Location b. Discharge and odor c. Edema and approximation d. Subinvolution Ans: C an episiotomy should be assessed for edema and approximation of incision. An edematous perineum causes more tension of d suture line and increased pain. Although d sutures may be difficult to visualize, the suture line should be intact. Episiotomy location is important, but not as important as the presence of edema. Discharge and odor refer to an assessment of lochia. Subinvolution refers to the complete return of the uterus to its prepregnancy size and shape. 18. In performing a routine fundal assessment, the nurse finds that the clients fundus is boggy. The nurse should first: a. Call the physician b. Massage the fundus c. Assess lochia flow d. Obtain an order for methylergonovine Ans: B the nurse should begin to massage the uterus so that it will be stimulated to contract. Assessing lochia flow can be done while the uterus is being massaged. The nurse shouldnt leave the client to call the physician. If the fundus remains boggy and the uterus continues to bleed, the nurse should use the call button to ask another nurse to call d physician. Methylergonovine may be prescribed, if needed. 19. Which type of lochia should the nurse expect to find in a client who is 2 days postpartum? a. Foul smelling b. Serosa c. Alba d. Rubra Ans: D lochia rubra lasts about 4 days followed by lochia serosa, which extends through the 7th day, and then lochia alba, which occurs during the 2nd and 3rd postpartum weeks. Foulsmelling lochia s a sign of infection. 20. A client treated with magnesium sulfate during labor is now on the postpartum unit. The nurse should be aware that the client is at risk for which of the following complications of magnesium sulfate therapy? a. Hypotension b. Uterine infection c. Postpartum hemorrhage d. Postpartum depression Ans: C because magnesium sulfate produces a smooth muscle depressive effect, the uterus should be assessed for uterine atony. The uterus may be unable to maintain a firm tone, thus increasing the risk of postpartum hemorrhage. Uterine infection and postpartum depression arent associated with magnesium sulfate therapy. Magnesium sulfate does decrease blood pressure, but its considered more of ananticonvulsant drug than an antihypertensive drug. 21. The nurse is assessing an infant with tracheoesophageal fistula. Which finding would the nurse expect to encounter? a. Increase in saliva b. Gastric tube easily passed c. Feeding without difficulty d. Normal chest x-ray Ans: A d infants inability to swallow saliva leads to an increase in saliva. The other options arent likely findings in tracheoesophageal fistula. The infant is unable to pass a gastric tube. During feedings, the infant is at risk for choking and cyanosis. Pulmonary infiltrates, labor collapse, and atelectasis frequently appear on the chest x ray. 22. A client is scheduled for amniocentesis. When preparing her for the procedure, the nurse should: a. Ask her to void b. Instruct her to drink 1 liter of fluid c. Prepare her for IV anesthesia d. Place her on her side

Ans: A to prepare a client for amnioceptesis, the nurse should ask d client to empty her bladder to reduce the risk of bladder perforation. The nurse may instruct the client to drink 1 L of fluid to fill d bladder before transabdominal ultrasound (unless ultrasound is done before amniocentesis to locate the placenta). I.V. anesthesia isnt given for amniocentesis. The client should be supine during the procedure; after ward, she should be placed on her left side to avoid supine hypotension, promote venous return, and ensure adequate cardiac output. 23. Six hours after birth, a neonate is transferred to the nursery. The nurse is planning interventions to prevent hypothermia. What is a common source of radiant heat loss? a. Low room humidity b. Cold weight scale c. Cool incubator walls d. Cool room temperature Ans: C common sources of radiant heat loss include cool incubator walls and windows. Low room humidity promotes evaporative heat loss. When the skin directly contacts a cooler object, such as a cold weight scale, conductive heat loss may occur. A cool room temperature may lead to convective heat loss. 24. A client is in the 25th week of pregnancy. Which procedure is used to detect anomalies? a. Amniocentesis b. Chorionic villi sampling c. Fetoscopy d. Ultrasound Ans: D ultrasound is used between 18 and 40 weeks gestation to identify normal fetal growth and detect fetal anomalies and other problems. Amniocentesis is done during the 3rd trimester to determine fetal lung maturity. Chorionic villi sampling is performed at 8 to 12 weeks gestation to detect genetic disease. Fetoscopy is done at about 18 weeks gestation to observe the fetus directly and obtain a skin specimen or blood sample. 25. Which nursing intervention has priority when feeding an infant with a cleft lip or palate? a. Directing the flow of milk in the center of the mouth b. Providing frequent, small feedings c. Avoiding breastfeeding d. Infrequent burping Ans: B frequent small feedings help to prevent fatigue and frustration in the infant. The flow of milk should be directed to side of the mouth. Breastfeeding may be possible. These infants need frequent burping because of d large amount of air swallowed while feeding. 26. During physical examination, a client in her 32nd week of pregnancy becomes pale, dizzy and lightheaded while supine. Which intervention takes priority? a. Turning the client onto her left side b. Asking the client to breathe deeply c. Listening to fetal heart tones d. Measuring the clients blood pressure Ans: A - as the uterus enlarges, pressure on the inferior vena cava increased, compromising venous return and causing blood pressure to drop. This may lead to syncope and other symptoms when the client is supine. Turning the client onto her left side relieves pressure on the vena cava, restoring normal venous return and blood pressure. Deep breathing wouldnt relieve this clients symptoms. Listening to fetal heart tones and measuring the clients blood pressure dont provide relevant information. 27. A client has meconium-stained amniotic fluid. The fetal monitor strip shows fetal bradycardia. Fetal blood sampling indicates a pH of 7.12. Based on this finding, which nursing intervention is called for? a. Administer oxygen, as prescribed b. Prepare for cesarean delivery c. Reposition the client d. Start IV oxytocin infusion, as prescribed Ans: B fetal blood pH of 7.19 or lower signals severe fetal acidosis; meconiumstained amniotic fluid and bradycardia are additional signs of fetal distress that warrant cesarean delivery. Oxygen administration and

client repositioning may improve uteroplacental perfusion but are only temporary measures. Oxytoxin administration increased contractions, exacerbating fetal stress. 28. Which of the following phases of uterine contractions is described as the letting-down phase? a. Increment b. Decrement c. Acme d. Variability Ans: B decrement is the lettingdown phase of uterine contractions. Increment refers to the buildingup phase, and acme is the peak of the contraction. Variability refers to the normal variation in the heart rate, caused by continuous interplay other parasympathetic and sympathetic nervous systems. 29. Which diagnostic procedure will best determine whether a client in labor has spontaneous rupture of amniotic membranes? a. Complete blood count b. Fern test c. Urinalysis d. Vaginal examination Ans: B a fern test indicates spontaneous rapture of amniotic membranes. The name of this test refers to the microscopic fernlike pattern produced by sodium chloride crystallization in dried amniotic fluid, which indicates the presence of ruptured amniotic membranes. A complete blood count might indicate infection (if white blood cells are increased), but it wont indicate whether the amniotic sac had ruptured. Urinalysis doesnt test for d presence of amniotic fluid. A vaginal examination may determine whether the membranes have ruptured but isnt conclusive. 30. A client is admitted to the hospital in preterm labor. To halt her uterine contractions, the nurse expects to administer: a. Magnesium sulfate b. Dinoprostone c. Ergonovine maleate d. Terbutaline Ans: D terbutaline, a beta 2receptor agonist, is used to inhibit preterm uterine contractions. Magnesium sulfate is used to treat pregnancyinduced hypertension. Dinoprostone is used to induce fetal expulsion and promote cervical dilation and softening. Ergonovine maleate is used to stop uterine blood flow, for example, in hemorrhage. 31. A 2-year-old child is admitted to the hospital with Hirschprungs disease. During the nursing history, the mother describes the childs stools to the nurse as foul-smelling and: a. Small, hard pebbles b. Large and frothy c. Cordlike d. Ribbonlike Ans: D choices A and C are not characteristic of Hirschprungs disease. Choice B is characteristic of cystic fibrosis. Ribbonlike stool pattern is characteristic of agangionic colon. 32. The nurse explains to a toddlers parents that the treatment of choice for congenital aganglionic megacolon would be: a. Surgical removal of affected colon b. Modified diet high in fiber c. Medication to stimulate the colon d. Permanent colostomy Ans: A the aganglionic section of the colon is removed so the remaining intestines can function. Diet changes will not make a difference owing to the lack of peristalsis. There is no medication that will make an aganglionic colon function. A permanent colostomy is not necessary. A temporary colostomy is performed using a two or three stage procedure to correct the problem.

33. A 7-year-old child is admitted to the hospital with nephritic syndrome. In the assessment phase, the nurse is aware that a classic symptom is: a. Increased urine output b. Hematuria c. Elevated blood pressure d. Proteinuria Ans: D there is decreased urine output. Hematuria is positive in glomerulonephritis. The blood pressure is normal or slightly below normal. There is a massive proteinuria. 34. During the edematous phase of nephritic syndrome, an important nursing intervention is to: a. Provide meticulous skin care b. Encourage fluid intake c. Encourage moderate activity d. Weigh the client every other day Ans: A edema increases the potential for skin breakdown, so skin care is extremely important. Fluid intake is limited to decrease the workload on the circulatory system with the excess fluid. The child should e weighed at least daily and often twice a day. 35. In evaluating the effectiveness of the prednisone therapy, the nurse realizes that a child with nephritic syndrome will continue to take the drug until after: a. Edema has disappeared b. Urine no longer contains protein c. Hematuria has resolved d. His moon face has disappeared Ans: B some edema may continue even after the drug has been stopped. Prednisone is continued as long as there is protein in the urine. Hematuria is a symptom of glomerulonephritis and not nephrosis. His moon face is a side effect of the drug and will continue as long as prednisone is taken.

36. A mother brings her 3-year-old son to the emergency room. He is crying with apparent acute abdominal pain. After initial assessment, intussusception is suspected. What type of characteristic stool would the mother most likely report? a. Black tarlike b. Ribbonlike c. Red currant-jellylike d. Frothy and foul-smelling Ans: C choice A would indicate old blood in the stool. Choice B is characteristic of Hirschprungs disease. Choice C is characteristic of intussusception and indicates fresh blood. Choice D is characteristic of cystic fibrosis. 37. A one-moth-old infant is at the physicians office for a follow-up visit after surgery for pyloric stenosis. Which of the following is the best indicator that the infant is recovering well from his surgery? a. Mother reports infants feeding well every 4 hours b. The infant has demonstrated a satisfactory weight gain c. The infant is in the 90% in length on the growth chart d. Mother reports infant has a normal stool pattern Ans: B choice A is subjective information and therefore not the best answer. Choice B is objective information that indicates the infant is maintaining and absorbing his feedings. Choice C is not directly related to food absorption. Choice D is subjective information and not the best indicator of food intake and absorption. 38. A baby has died from sudden infant death syndrome (SIDS). SIDS is often initially mistaken for: a. Failure to thrive b. Viral infection c. Meningitis d. Child abuse Ans: D choice A, B and C and conditions that have no symptoms that could be mistaken for SIDS. Bruising

occurs due to the pooling and settling of blood once the infant has died. This gives the appearance that the child has been beaten. 39. During the initial assessment of a child with Reyes syndrome, the mother reports that about a week ago, the child had: a. Mumps b. Meningitis c. Influenza d. Cellulites Ans: C choices A, B and D are conditions not associated with Reyes syndrome. Influenza usually precedes Reyes syndrome. 40. The most important nursing intervention in caring for a child with Reyes syndrome is to: a. Prevent skin breakdown b. Monitor intake and output c. Do range-of-motion exercises d. Turn every 2 hours Ans: B this is not a life-threatening problem. Careful monitoring of intake and output aids in preventing cerebral edema or dehydration. Choice c is not associated with a life-threatening problem. This intervention is not as important as preventing cerebral edema or dehydration. 41. Because of liver involvement associated with Reyes syndrome, the nurse should use which special caution when caring for children with this condition: a. Administering IM injections b. Monitoring output from the catheter c. Assessing the level of consciousness d. Turning the child Ans: A prolonged bleeding may occur owing to impaired coagulation. Pressure should be applied to the injection site for a longer period of time. Choices B, C and D are not related to liver function. 42. A one-year-old infant is admitted to the hospital to rule out cystic fibrosis. During the admission process the infant passes a stool. The nurse realizing that his stool is symptomatic of cystic fibrosis, charts it as: a. Small and constipated b. Green and odorous c. Large and bulky d. Yellow and loose Ans: C choice A and D type of stool are not symptomatic of CF. Stools are not green but are foul smelling. Nondigested food, owing to malabsorption, is excreted, causing an increase in amount and bulk of stools. 43. A child is diagnosed with cystic fibrosis. He is receiving pancreatic enzymes. Once the pancreatic enzymes the child is taking are effective, he will: a. Have normal bowel movements b. Increase 2 lb in weight per week c. Have decreased NaCl in his sweat d. Have fewer respiratory infections per year Ans: A pancreatic enzymes aid in absorption of nutrients from the intestines so the stools become normal. Choice B is not a realistic weight gain. Pancreatic enzymes are not related to the respiratory system. Pancreatic enzymes are not related to the respiratory system. Pancreatic enzymes are not related to the NaCl level in the sweat. 44. A 6-year-old is hospitalized with acute lymphocytic leukemia. She is placed on protective isolation, which concerns her parents. The nurse should explain that this will: a. Protect her from too many visitors b. Protect her from infectious organisms c. Provide a quiet, private environment for her d. Keep other children away from the child

Ans: B the purpose of protective isolation is to protect the child from exposure to organisms from other people. With leukemia, changes in the blood cell number and composition make the child susceptible to infection. The purpose of protective isolation is to protect the child from exposure to organisms from other people. Choice D is not the purpose of protective isolation. Preventing infection through direct contact with anyone is the purpose. 45. The nurse discusses mouth care with a 6-year-old girl who has acute lymphocytic leukemia and her mother. The nurse explains that when tooth-brushing is contraindicated, the most effective way to clean teeth is: a. Rinsing with water b. Chewing gum after eating c. Rinsing with hydrogen peroxide d. Use a Water Pik Ans: D choice A is not very effective and does not stimulate the gums. Choice B is not very effective and does not stimulate the gums. Choice C does not stimulate the gums and prevent gingivitis. Choice D will effectively rinse the mouth and stimulate the gums. 46. A 12-year-old girl hospitalized with a diagnosis of rheumatic fever. To minimize her joint pain during acute episodes, the nurse should teach the parents to: a. Immobilize the joints in a functional position b. Do full range of motion on all joints daily c. Apply heat to the involved joints d. Massage joints briskly with lotion after her bath Ans: A immobilization allows rest and healing. The pain can be so intense that even the weight of a blanket can hurt. Movement of joints (choices B and D) causes pain. The pressure of the healing pad or hot water bottle can cause pain. 47. Discharge planning of a child with rheumatic fever should include teaching the child and parents to recognize which of the following toxic symptoms of sodium salicylate? a. Blurred vision and itching b. Chills and fever c. Acetone breath odor d. Tinnitus and nausea Ans: D Choice A, B and C are not toxic symptoms. Choice D are common toxic symptoms of salicylates. 48. A neonate in the newborn nursery is suspected of having a tracheoesophageal fistula. A major symptom the nurse observed was: a. Hypersensitive gag reflex b. Dry mouth with no drooling c. Cyanosis d. Lethargy Ans: C Choice A and D are not symptoms of this condition. TEF symptoms are excessive salivation and drooling. Cyanosis is due to the fistula from the trachea and the esophagus. 49. A 3-year-old child is diagnosed with Kawasakis disease. The nurse observes which of the following symptoms? a. Below-normal temperature b. Strawberry tongue c. Edema of the face d. Swelling in the groin Ans: B with this disease, there is fever for more than 5 days. Strawberry tongue is a symptom of the disease. There is also reddening of the rest of the oropharynx. There is edema of the hands and feet as well as redness. Swelling occurs in the cervical lymph nodes with this disease. 50. The therapeutic management for a child who has been diagnosed with Kawasakis disease will include administering which of the following medications?

a. Acetaminophen (Tylenol) b. Globulin c. Antibiotics d. Steroids Ans: B aspirin is usually given to reduce inflammation. Globulin is given to minimize possible cardiac complications. Antibiotics and steroids are not usually given for this condition. 51. A mother is being instructed on the best method of administering syrup of ipecac in the initial home management of an accidental ingestion. The nurse should inform her that syrup of ipecac should be administered with: a. Milk b. Activated charcoal c. One to two glasses of tepid water d. Large amounts of cold water Ans: C milk or carbonated drinks should be avoided with administration of syrup of ipecac because they may delay emesis. The purpose of administering activated charcoal is to bind with the poison so that body absorption of the poison will be decreased. Because the purpose of syrup of ipecac is to induce vomiting, these drugs would decrease effectiveness of each other. The therapeutic action of S of I is facilitated by following the dose with 100 to 200 ml of tepid water or other clear liquids in children (200 300 ml in adults). There could be a problem with water intoxication and decreased effectiveness of S of I with the administration of large amounts of cold water. 52. A 7-year-old child has been taking phenytoin and Phenobarbital for control of chronic recurrent seizures. In the physical exam, the nurse notes that the child has hyperplasia of the gums. The nurse should recognize that hyperplasia of the gums is: a. A common occurrence with chronic recurrent seizures b. A common side effect of phenytoin c. Not related to the drugs or the disease d. An unusual side effects of phenobarbital Ans: B many children who have seizures do have this side effect. However, it has nothing specifically to do with the seizure. Hyperplasia of the gums is a side effect of phenytoin. Phenytoin administration is seen most commonly in children and adolescents. It never occurs in edentulous clients. Hyperplasia of the gums is a side effect of phonation. Phenytoin administration is seen most commonly in children and adolescents. It never occurs in edentulous client. Hyperplasia of the gums is not a side effect of Phenobarbital. 53. A 7-month-old infant has been on antibiotic therapy. The nurse notes that the child has white patches in his mouth that will not rub off. The physician orders nystatin (Mycostatin) 1 ml, PO, QID. The nurse should realize that the appropriate technique in administering this medication is to: a. Give 0.5 ml in each side of the mouth b. Give with milk or food c. Give through a nipple d. Follow with water Ans: A nystatin is a local antibiotic and must come into contact with the infected area to be effective. Giving half of the dose on each side of the mouth will increase the area of contact and consequently increase the effectiveness of the drug. Giving with milk or food, through a nipple, or following medication with water (choices B, C and D) would decrease effectiveness by decreasing direct contact of the medication with the infected area. 54. A primary objective for planning nursing care of an edematous child with nephritic syndrome would be to: a. Ambulation b. A low-carbohydrate diet c. A high-protein diet d. A low-protein diet Ans: C the child with nephrosis should be on bed rest in the edematous state. High-carbohydrate diet is needed for energy and the caloric intake. Protein replacement is critical because of the massive proteinuria and hypoalbuminemia with nephrosis. High=protein diet is needed for protein replacement.

55. The predominant purpose of the first APGAR score of a newborn is to: a. Determine gross abnormal motor function b. Obtain a baseline for comparison with the infants future adaptation to the environment c. Evaluate the infants vital function d. Determine the extent of congenital malformation Ans: A- Apgar scores are not related to the infants care, but to the infants physical condition. (B) Apgar scores assess the current physical condition of the infant and are not related to future environmental adaptation. (C) the purpose of the Apgar system is to evaluate the physical condition of the newborn at birth and to determine if there is an immediate need for resusciation. (D) congenital malformations are not one of the areas assessed with Apgar scores. 56. Provide the one-minute APGAR score for an infant born with the following findings: respiratory effort, slow and irregular; muscle tone, some flexion of extremities; reflex irritability, vigorous cry and; color, body pink, blue extremities. a. 7 b. 10 c. 8 d. 9 Ans: A- seven out of possible perfect score of 10 is correct. Two points are given for heart rate above 100; 1 point is given for slow, irregular respiratory effort; 1 point is given for some flexion of extremities in assessing muscle tone; 2 points are given for vigorous cry in assessing reflex irritability; 1 point is assessed for color when the body is pink with blue extremities (acrocyanosis). (B) for a perfect Apgar score of 10, the infant would have a heart rate over 100 but would also have a good cry , active motion, and be completely pink. (C) for an Apgar score of 8 the respiratory rate, muscle tone, or color wouldneed to fall into the 2 point rather than the 1point category. (D) for this infant to receive an Apgar score of 9, four of t he areas evaluated would need ratings of 2 points and one area a rating of 1 point 57. An 8-year-old child comes to the physicians clinic complaining of swelling and pain in the knees. His mother says, The swelling occurred for no reason, and it keeps getting worse. The initial diagnosis is Lyme disease. When talking to the mother and child, questions related to which of the following would be important to include in the initial history? a. A decreased urinary output and flank pain b. A fever over 103oF occurring over the last 2 3 weeks c. Rashes covering the palms of the hands and the soles of the feet d. Headaches, malaise or sore throat Ans: A- urinary tract symptoms are not commonly associated with Lyme disease. (B) a fever of 103.oF is not characteristic of lYme disease. (C) the rash that is associated with Lyme disease does not appear on the palms of the hands and the soles of the feet. (D) classic symptoms of Lyme disease include headache, malaise, fatigue, anorexia, stiff neck, generalized lymphadenopathy, splenomegaly., conjunctivitis, sore throat, a abdominal pain, and cough 58. The most commonly known vectors of Lyme disease are: a. Mites b. Fleas c. Ticks d. Mosquitoes Ans: A- mites are not common vector of Lyme disease. (B) fleas are not the common vector of Lyme disease (C) ticks are the common vector of lyme disease. (D) mosquitos are not the common vector of Lyme disease 59. A specific laboratory technique specific for diagnosing Lyme disease is: a. Polymerase chain reaction b. Heterophil antibody test c. Decreased serum calcium level d. Increased serum potassium level Ans: A-nursing process phase; analysis; client need; physiological integrity area: pediatrics polymerase chain reaction is the laboratory technique specific for lyme disease (B) heterophil antibody test is used to diagnose mononucleosis. (C) lyme disease does not decrease the serum calcium level. (D) lyme disease

does not increase the serum potassium level 60. The nurse would expect to include which of the following when planning the management of the client with Lyme disease? a. Complete bed rest for 6-8 weeks b. Tetracycline treatment c. IV amphotericin B d. High-protein with limited fluids Ans: A- the client is not placed on complete bed rest for 6 weeks. (B) tetracycline is the treatment of choice for children with Lyme disease who are over the age of 9. (C) IV amphotericin B is the treatment for histoplasmosis. (D) the client is not restricted to a high-protein diet with limited fluids 61. A six-month-old infant has been admitted to the emergency room with febrile seizures. In the teaching of the parents, the nurse states that: a. Sustained temperature elevation over 103oF is generally related to febrile seizures b. Febrile seizures do not usually recur c. There is little risk of neurological deficit and mental retardation as sequele to febrile seizures d. Febrile seizures are associated with disease of the central nervous system Ans: A-the temperature elevation related to febrile seizures generally exceeds 101oF, and seizures occur during the temperature rise rather than after a prolonged elevation. (B) febrile seizures may recur and are more likely to do so when the first seizure occurs in the 1st year of life. (C) There is little risk of neurological deficit mental retardation, or altered behavior secondary to febrile seizures. (D) Febrile seizures are associated with disease of the central nervous system

General Nursing Board 101 PREBOARD Nursing Practice 2 http://nsgbrd101.proboards.com/index.cgi?board=answers&action=display&thread=18 PREBOARD Nursing Practice 2 Post by admin on Aug 18, 2010, 1:39pm

PAGE IS STILL UNDER CONSTRUCTION-----------------1. The following signs for placental separation are the following EXCEPT: a. The uterus rises b. A sudden gush of blood c. The uterus becomes round in shape d. Cord gets longer and the fundus of the uterus is going far from the navel Ans. D. The fundus of the uterus will be nearer or just above the mothers navel not far from the navel. 2. When the cervix is dilated for about 0-3 cm and contractions are weak, less than 2 to 10 minutes. What stage of labor is this? a. first stage c. second stage b. third stage d. fourth stage Ans. A. first stage 3. This normal change in the delivering mother is characterized by mild mood disturbances, marked by emotional instability. This is especially true to first time mothers. a. Postpartum depression b. Postpartum blues

c. Postpartum psychosis d. None of the above Ans. B. Postpartum blues. Most women may feel very sad and overwhelmed after they have a baby. Postpartum blues are caused by changes in womans body during pregnancy and changes in her lifestyle after childbirth. ( 4. The following are signs or puerperal sepsis EXCEPT: a. Fever on any two of the first 10 days post partum b. Absence of pelvic pain and abdominal tenderness c. Abnormal lochia with foul smell odor d. all of the above Ans. B. Presence of pelvic and abdominal tenderness is one sign of puerperal sepsis. 5. Possible puerperal sepsis of the genital tract occurs during which of the following for the first time? a. during rupture of the bag of water b. during the placental separation c. during the cervical effacement d. during the crowning Ans. A. Infection of the genital tract happening at any time between rupture of the bag of waters (or start of labor) up to the 42nd day post partum. 6. The following BUT one are the noticeable signs of pre eclampsia that needs prompt interventions: a. Severe headache b. Epigastric Pain c. Blurring of Vision d. None of these Ans. D. None of these. Choices a,b and c are the real manifestations and signs of pre eclampsia that should be referred to the hospital urgently. 7. During a postpartum period where there is profuse bleeding, urgent action must be taken by the nurse. The following are the most important procedures EXCEPT: a. Compression of the aorta b. Bimanual Compression of the uterus c. B only d. except C Ans. D. Both the Compression of the Aorta and Bimanual Compression of the uterus are important during period of profuse bleeding. Remember to prepare to go or already be on the way to the hospital with the patient in case this life saving procedures are performed. 8. Which is TRUE of Oxytoxin? a. Sustained (tonic) uterine contraction b. Onset 6-7 minutes c. Common side effects are hypertension and vomiting d. Rythmic uterine contractions Ans. D. Oxytoxins action to the uterus is rhythmic contractions with onset of 2-3 minutes and side effects of hypotension. ( 9. It is a first line contraceptive method for postpartum women. One will benefit the baby and the other for contraception. a. Combined estrogen-progesterone contraceptive b. Lactational Amenorrhea Method c. Barrier Method d. Female sterilization Ans. B. The lactational Amenorrhea Method (LAM) is a first line contraceptive method for postpartum women. Breastfeeding benefits the baby by providing the best nutrition. It also provides an effective method of contraception. 10. Julian decided to use a contraceptive method, she delivered her first born baby Carlisle 1 month ago. She breastfeeds her baby. When asking the nurse for the first choice of contraception regarding her situation what will be the best response by the midwife? a. You can use Progestin only pills since this will not affect a womans milk supply. b. Well, combined oral contraceptive will be favorable since this contains estrogen.

c. I suggest Condom since this can be used immediately and definitely will not interfere with breastfeeding. Safe to use any time after birth. d. You cant use any contraceptive at this time, you need at least 2 months for that. Ans. C. First Choice: LAM, Condom, BTL, IUD, Tubal Ligation and Vasectomy Can be used immediately, Does not interfere with breastfeeding and safer to use any time after birth; Second Choice: Progestin only pills, Progestin only injectables and Progestin implants- Contain Progestin, Dont affect womans milk supply. Recommended to wait six weeks after delivery before using a progestin only method. Third Choice: Combined Oral Contraceptives and Combined Contraceptive patch- Contain estrogen, which can reduce a womans milk supply. Advise to delay the use of such methods until after six months postpartum. 11. Which program broadens content on womens health and safe motherhood? a. Reproductive health b. Family planning program of the Philippines c. Maternal and child health nursing d. Maternity programs Ans. A. Reproductive Health 12. It is a method by identifying the fertile and infertile days of the menstrual cycle as determined through a combination of observations made on the cervical mucus, basal body temperature recording and other signs of ovulation: a. Two day method b. Sympto-thermal method c. Standard days method d. Lactating Amenorrhea Method Ans. B. ( 13. The nursing goal during delivery is: a. Safety of the mother and child b. Prevention of complications for both mother and child c. Safe and efficient delivery d. all of the above Ans. A. Safety of the mother and child 14. Breastfeeding is started 30 minutes after normal delivery for: a. uterine contraction b. bonding c. financial reason d. contraception Ans. A. Uterine contraction 15. A lactating mother is scheduled for an out-of-town seminar and she asked the nurse for an advice how she can continue her breastfeeding. The BEST response is: a. Dont you worry, you can always bottle-feed your baby in your absence. b. You can do milk banking by saving enough milk and put it in the freezer for storage c. Bring the baby with you to insure that the baby is well cared for d. none of the above Ans. B. You can do milk banking by saving enough milk and put it in the freezer for storage 16. Which of the following women should be considered as special targets for family planning? a. Those who have two children or more b. Those with medical condition such as anemia c. Those younger than 20 years and older than 35 years d. Those who just had a delivery within the past 15 months Ans. D. Those who just had a delivery within the past 15 months 17. Freedom of choice is one of the policies of the Family Planning Program of the Philippines. Which of the following illustrates this principle? a. Information dissemination about the need for family planning b. Support of research and development in family planning methods c. Adequate information for couples regarding the different methods d. Encouragement of couples to take family planning as a joint responsibility Ans. C. Adequate information for couples regarding the different methods 18. During prenatal consultation, a client asked you if she can have her delivery at home. After history taking and physical examination, you advised her against a home delivery. Which of the following finding

disqualifies her for a home delivery? a. Her OB score is G5P3 b. She has some palmar pallor c. Her blood pressure is 130/80 d. Her baby is in cephalic presentation Ans. A.Her OB score is G5P3 19. You are in the clients house to attend a delivery. Which of the following will you do first? a. Set up a sterile area b. Put on a clean gown and apron c. Cleanse the clients vulva with soap and water d. Note the interval, duration and intensity of labor and contractions Ans. D. Note the interval, duration and intensity of labor and contractions 20. In preparing for a primigravida for breastfeeding, which of the following will you do first? a. Explain to her that putting the baby to breast will lessen blood loss after delivery b. Instruct her to wash her nipples before and after each breastfeeding c. Teach her nipple stretching exercises if her nipples are erect d. Tell her that lactation begins within a day after delivery Ans. D. Tell her that lactation begins within a day after delivery 21. In a mothers class you discuss proper breastfeeding technique. Which of these is a sign that the baby has latched on the breast properly? a. The baby takes shallow, rapid sucks b. The mother does not feel nipple pain c. The babys mouth is only partly open d. Only the mothers nipple is inside the babys mouth Ans. B. The mother does not feel nipple pain 22. In the postpartum home visit, the nurse should do the following, EXCEPT: I. Do cord inspection II. Palpate the fundus and the fetal heart beat III. Bathe the baby IV. Discuss the benefits of family planning a. I and II b. II and IV c. I and III d. All except II Ans. D. All except II 23. All of the following are activities of the nurse during pre-consultation conference I. Urine examination II. Taking the BP and weight III. Leopolds maneuver IV. Write the findings on the clients record V. Explain the needed care a. I,II and III c. III,IV and V b. I, II and IV d. I, III and V Ans. B. I, II and IV 24. The APGAR score assesses the babys color, heartbeat, reflexes, muscle tone and breathing. In breathing, how many breaths cycle per minute when the midwife considers it as very rapid breathing? a. less than 30 per minute b. more than 60 per minute c. less than 60 per minute d. more than 30 minute Ans. B. very rapid breathing more than 60 breaths cycle per minute; very slow breathing less than 30 breaths cycle per minute. 25. During the initial assessment of the new born the nurse found out that the baby is breathing inadequate and the babys appearance and color is blue. What is the immediate responsibility of the nurse? a. Dry and wrap the baby and the baby stays with the mother b. Dry and wrap the baby and clear the airway c. Refer to the doctor d. A and B Ans. B.

INITIAL ASSESSMENT ACTION Pink, Heart Rate>120 bpm, Breathiing regularly Dry and wrap the baby Baby stays with the mother Blue, Heart Rate >100bpm, Breathing Inadequate Dry and wrap Clear the airway Blue or pale, Heart Rate <100 bpm, Not breathing Dry and wrap Clear the airway Ask for help Refer to the doctor 26. The initial assessment shows Pink, Heart Rate >120 bpm and breathing regularly the priority action is: a. Ask for help b. Dry and wrap the baby and let the baby stay with the mother c. Dry and wrap the baby and clear the airway d. Dry and wrap the baby and refer to the doctor Ans. B. INITIAL ASSESSMENT ACTION Pink, Heart Rate>120 bpm, Breathiing regularly Dry and wrap the baby Baby stays with the mother Blue, Heart Rate >100bpm, Breathing Inadequate Dry and wrap Clear the airway Blue or pale, Heart Rate <100 bpm, Not breathing Dry and wrap Clear the airway Ask for help Refer to the doctor 27. Which of the following is NOT true of colostrum? a. It has powerful antibodies. b. It will fight against infections. c. It makes the baby strong. d. It promotes mother-baby bonding. Ans. D. Breast feeding promotes mother-baby bonding and the touching makes babies feel more secure and emotionally stable. Colostrum has a powerful antibodies to fight against infection and make the baby strong. 28. Marieta complains of cracked nipples, the nurse implemented her interventions however she wrongly intervened when she said to the mother: a. to continue breastfeeding b. to air dry her nipples, leaving a little breast milk on the nipple after nursing c. to wash her nipple with water and soap d. to position the baby so that the areola is completely in the babys mouth Ans. C. 29. Mavericks handles a case of postpartal mother and family focusing on home care. He needs to schedule a first visit to OB client Leila on: a. within 1 week after discharge b. within 24 hours after discharge c. within 1 hour after discharge d. within 1 month after discharge Ans. A. The first postpartal home visit after discharge is done within one week after discharge. The first home visit after home delivery is done within 24 hours. 30. If you were Maverick, which of the following actions would alert you that a new mother is entering a postpartal taking-hold phase? a. She sleeps as if exhausted from the effort of labor b. She urges the baby to stay awake so that she can breast feed him on her c. She says that she has not selected a name for the baby as yet d. She tells you she was in lot pain all during labor Ans. B. During the taking-hold phase, the woman assumes her responsibilities to her new child. All other options are usually characteristics of the taking in phase

Situation 7: Knowledge of the concepts and processes of CHN is important 31. The concern of the CH nurse is not only those who go to the center but also those who do not. What is the reason for this? A. determine the reasons why people do not go to the center B. identify health needs of people who do not come C. define the needs and problems of those who come D. ensure that CH services are comprehensive 32. Which of the following statements is correct? A. Cognizant of the devolution of health services, decision-making on health affairs should be made by the office of the mayor at the municipal level. B. Given a community, the nurse can uplift the situation by herself. C. Health and illness have the same meanings in a homogenous community. D. While health is a responsibility of the health department, it is as much responsibility of the people themselves. 33. To facilitate implementation of a CHN project, the following are to be considered EXCEPT A. clear tasks and responsibilities B. limited involvement of community members C. specific objectives with measurable output D. available materials and supplies 34. When promoting the health of the individual, family and community, the first step to take by the CH nurse is A. establish a registry of families C. do a health assessment B. conduct a community census D. organize the community 35. The nurse makes a continuous appraisal of the care she provides for the community in relation to the objectives of care and the response of the community. This process is called community health A. assessment B. diagnosis C. participation D. evaluation Situation 8: The Health Situation of the Philippines 36. The basis of poor health situation in the country is A. poverty C. low priority on health B. lack of government support D. graft and corruption 37. If infant mortality rate is high, emphasis should be given to A. communicable disease control C. maternal and child health B. nutrition program D. immunization program 38. The following contribute to poor health situation of the majority 1 - high cost of health services 3 - crisis-oriented concept of health 2 - low priority for health 4 - backward economy A. 1 only B. 1, 2 and 3 C. 1, 3 and 4 D. all of these 39. The following statements describe the health human resources EXCEPT A. Every year, there is a marked increase in number of health professionals. B. Despite of the increase in number of health providers, there is still an enormous need in the countryside. C. The growing number of health professionals contributes to their low salaries and lack of benefits. D. Migration to other countries is solely an economic issue. 40. Despite of the preventive nature of the diseases comprising morbidity and mortality patterns, hospitals has been developed and maintained as primary venues of health services because 1 - diseases are far advanced and require hospitalization 2 - not all diseases can be prevented at the community level 3 - hospitals are infrastructures that project development 4 - hospital practice is rooted from colonial-oriented health science education A. 1 and 2 B. 1, 2 and 3 C. 2 and 3 D. 1, 2, 3 and 4 Situation 9: Health promotion 41. The goal of health promotion is A. identify and minimize risk factors C. popularization of healthy habits

B. rehabilitation of sick individuals D. increase in wellness level 42. Which of the following pertains to health promotion? A. health protection manifesting avoidance behavior B. specific to a particular health problem C. approach activities to enhance health capabilities D. shorten the duration of the disease process 43. Individual wellness refers to A. the person's ability to abide by community norms B. person's maintenance of healthy lifestyle C. the person ability to adapt to his external environment D. the individuals formation of health beliefs and habits 44. In health promotion, community wellness is ensured through the promotion of A. healthy lifestyles C. healthy behavior and beliefs B. healthy environment D. healthy norms 45. Nutrition as an important method of health promotion is ensured through A. growth monitoring C. taking vitamins B. balanced diet D. micro-nutrient supplementation Situation 10: Disease Prevention 46. The best time for disease prevention is during this phase of disease process A. asymptomatic B. prepathogenic C. symptomatic D. pathogenic 47. Primary level disease prevention methods are applied during this phase A. asymptomatic B. prepathogenic C. symptomatic D. pathogenic 48. Secondary level disease prevention methods are applied in this phase A. asymptomatic B. prepathogenic C. symptomatic D. pathogenic 49. Tertiary level disease prevention is applied in this phase A. asymptomatic B. prepathogenic C. symptomatic D. pathogenic 50. A particular disease is far advanced if it is at this phase A. asymptomatic B. prepathogenic C. symptomatic D. pathogenic Situation 11: Activities in health promotion and disease prevention 51. Establishing youth clubs and mothers' organization in the community is A. primary level disease prevention C. tertiary level disease prevention B. secondary level disease prevention D. health promotion 52. Assistance in physical therapy of a trauma patient A. primary level disease prevention C. tertiary level disease prevention B. secondary level disease prevention D. health promotion 53. Conducting clinics in prisons to determine extent of infectious diseases is A. primary level disease prevention C. tertiary level disease prevention B. secondary level disease prevention D. health promotion 54. Giving lectures on sex education to high school students is A. primary level disease prevention C. tertiary level disease prevention B. secondary level disease prevention D. health promotion 55. Identifying sick individuals in the community for prompt treatment is A. primary level disease prevention C. tertiary level disease prevention B. secondary level disease prevention D. health promotion Situation 12: In CHN practice, people's participation is essential. 56. Community participation is made possible through

A. conducting consultation meetings with key leaders of the community B. full participation of nurses in decision-making C. provision of comprehensive and understandable information D. shared leadership in the community at the onset of the program 57. To effectively provide nursing care to the community, the CH nurse must first A. understand community's culture C. identify community leaders B. assess community beliefs D. review existing records in health 58. The minimum level of genuine people's participation is A. consultation B. shared leadership C. self-reliance D. placation 59. In addressing the health problems of the community, CH nurse ensures A. people identify and prioritize their own problems B. each health team member provides her/his share in health care C. people act as recipients of services D. DOH programs are being carried out 60. People's participation is made possible through A. primary health care C. community development B. multi-disciplinary approach D. community organizing Situation 13: In CHN, the nurse utilizes community organizing (CO) . 61. The goal of CO is A. community participation C. people's organization B. community development D. people's awareness

62. Preliminary social investigation utilizes A. primary data sources C. registries and records B. interviews and observation D. focus group discussions 63. Upon entry to the community, the nurse can start the following EXCEPT A. deepening social investigation C. small-group formation B. community integration D. social preparation 64. Manageable units of the community to facilitate service delivery and people's participation is called A. spot map B. small group C. core-group D. organizing group 65. The basic reasons why community organizers need to phase out from the community is to enable A. nurse to open CO work in other depressed communities B. people to exercise self-reliance C. the people's organization to expand their coverage D. people to test their unity and strength Situation 14: The primary health care (PHC) approach 66. The following statements pertaining to PHC are correct EXCEPT A. ultimate goal is self-reliance C. responds to immediate health problems B. alternative health system D. utilizes appropriate technology 67. PHC is effective when CH nurses A. are models of health C. initiate mass actions B. lead people in community mobilizations D. trust people's capabilities 68. The goal of PHC according to the Department of Health is A. health for all by the year 2000 B. health for all in 2000 and health in the hands of the people by 2020 C. health for all by 2000 and beyond D. provide equity and quality health care in partnership with people 69. For common coughs and colds, these medicinal plants can be used EXCEPT

A. lagundi B. sambong C. bayabas D. luya 70. Niog-niogan is best for A. coughs and colds B. diarrhea C. ascariasis D. asthma Situation 15: There are different levels of clientele in CHN. 71. An instrument for gathering subjective data from the individual is A. nursing history C. use of laboratory exam findings B. physical assessment D. process recording 72. The exact opposite of the current problematic situation is stated as a A. plan B. goal C. objective D. evaluation 73. In the assessment phase of the family health nursing process, an indicator for problem prioritization of family health problem is A. nature of the problem C. modifiability B. preventive potential D. salience 74. By its nature, which of the following problems will be given LEAST priority? A. unemployment B. scabies C. poor home environment D. ascariasis 75. Which of the following indicators are used for determining the family's value on prevention? 1 - updated immunization schedule 3 - compliance behavior 2 - family's concept of prevention 4 - eating habits A. 1 and 2 B. 1 and 3 C. 1, 2 and 3 D. 1, 2, 3 and 4 Situation 16: The Population Group 76. The constant presence of en etiologic factor and a disease in a particular area is called A. sporadic B. endemic C. epidemic D. pandemic 77. To establish new cases of a disease by studying those exposed not exposed to a risk factor, the nurse must conduct this study A. Case control B. cross-sectional C. prospective D. experimental 78. The primary aim of occupational health nursing is A. reduction of hazards in the work area B. maintenance of a healthy work environment C. promotion of optimum health and prevention of diseases and injuries D. industrial peace among workers and management 79. Workers may be exposed to different hazards of work like biological hazards exemplified by A. stress, tenure, salary C. heat, cold, humidity B. dust, fumes, chemicals D. fungi, virus, bacteria 80. To weed out drug users in the school, the nurse sets up a drug watch committee. This is an activity under what component of school health nursing? A. school health services C. healthful school living B. school health instruction D. school-community linkage Situation 17: Community Diagnosis 81. The most common method for community diagnosis that accounts for the bulk of data is A. census B. records review C. interview D. sample survey 82. The following are steps in processing the data 1 data analysis 3 data presentation 2 data collection 4 data collation A. 2, 4, 1 and 3 B. 2, 3, 4 and 1 C. 2, 4, 3 and 1 D. 2, 3, 1 and 4 83. Major factors affecting population include the following EXCEPT A. births B. deaths C. migration D. morbidity 84. A population pyramid with triangular shape and broad base indicates the following EXCEPT

A. high death rates B. poverty C. young population D. more females 85. The sampling method used for a reliable community diagnosis is A. simple random B. multi-stage C. cluster D. systematic random Situation 18: Vital Statistics 86. The crude birth rate of X Community is 30.5. This means A. 30.5 births in every 1000 population B. 31 births in every 1000 population C. 30 births in every 1000 population D. 30 births in 1000 population of women 15 to 44 years old 87. Infant mortality rate refers to deaths in every 1000 births belonging to A. 0 28 days B. 0 1 month C. 0 1 week D. 0 12 months 88. There were 20 cases of measles in a community and 5 died. What is the case fatality rate of measles? A. 0.025 B. 2.5 C. 25% D. 250 89. The relative importance of a cause of death is computed as A. case fatality rate C. proportionate mortality rate B. cause-of-death rate D. incidence rate 90. In estimating the rank of a cause as a common illness, the nurse makes use of A. incidence rate B. prevalence rate C. Swaroops index D. attack rate Situation 19: FP, EPI 91. The following statements refer to the family planning program EXCEPT A. The policy program advocates partnership, participation and promotion of health. B. The main aim of the program is birth control. C. The program is anchored on safe motherhood and child survival. D. Females may decide independently on choice of contraceptive method. 92. Which of the following statement is correct? A. Effectiveness or sterility will come after 25 to 40 days after vasectomy. B. Multiparous women have high fertility post-IUD removal. C. Pills are advisable to those who have occasional sex. D. Condoms can be reused. 93. Life-long immunity for tetanus is achieved by a woman after A. 3 pregnancies C. 3 booster doses B. 2 doses during pregnancy D. fourth tetanus toxoid 94. DPT is given at A. 0.05 ml. ID B. 0.5 ml IM C. 0.5 ml SQ D. 0.1 ml ID 95. Which of the following is a normal reaction to BCG vaccination? A. deep abscess C. glandular enlargement B. indolent ulceration D. Kochs phenomenon Situation 20: Nutrition, CARI, CDD 96. The following statements are true regarding kwashiorkor EXCEPT A. food is the only cure B. child is apathetic and does not want to eat C. characterized by skin sores, edema, and moon-facie D. usually starts when the child is less than one year old 97. The recipients of targeted food assistance program are A. general public C. pregnant women B. children D. children and pregnant women 98. Belen is one month old and has pneumonia. The intervention should be A. tepid sponge bath C. refer immediately

B. provide antibiotics and see after 2 days D. assess for other signs 99. The initial sign of dehydration in diarrhea is A. weight loss B. positive skin fold test C. comatose D. thirst 100. Ana is 1 year old and with 5 bouts of diarrhea in a day. What should be given at every bout? A. cup of rice water B. 1 glass oresol C. antibiotic D. IV fluids

You might also like